You are on page 1of 55

Table of Contents

Relation, Function & Inverse Trigonometric Functions

 Theory ............................................................................................................................................ 2

 Solved examples ............................................................................................................................ 11

 Exercise - 1 : Basic Objective Questions ................................................................................... 28

 Exercise - 2 : Previous Year JEE MAINS Questions ....................................................................... 34

 Exercise - 3 : Advanced Objective Questions ............................................................................ 39

 Exercise - 4 : Previous Year JEE Advanced Questions .................................................................. 49

 Answer Key .................................................................................................................................... 55


2 RELATION , FUNCTION & INVERSE TRIGONOMETRIC FUNCTIONS

RELATION, FUNCTION & INVERSE TRIGONOMETRIC FUNCTIONS

RELATION AND FUNCTION–II 1.2.3 Inverse of a relation : Let A, B be two sets and let R
be a relation from a set A to set B. Then the inverse of
–1
1. RELATION R, denoted by R , is a relation from B to A and is
defined by
–1
1.1 Cartesian product of sets R = {(b, a) : (a, b)  R}
–1
Clearly, (a, b)  R  (b, a)  R
Definition : Given two non-empty sets P & Q. The cartesian
–1 –1
product P × Q is the set of all ordered pairs of elements Also, Dom (R) = Range (R ) and Range (R) = Dom (R ).
from P & Q i.e. 1.2.4 Types of relation :
P × Q = {(p, q); p  P; q  Q} (a) Void Relation : Let A be a non-empty set. Then
 A × A and so it is a relation on set A. This relation
1.2 Relations
is called the void or empty relation on set A.
1.2.1 Definition : Let A & B be two non-empty sets. Then (b) Universal Relation : Let A be a non-empty set. Then,
any subset ‘R’ of A × B is a relation from A to B. A × A is known as the universal relation set A.
If (a, b)  R, then we write it as a R b which is read as (c) Identity Relation : Let A be a non-empty set. Then, IA
‘a is related to b’ by the relation R, ‘b’ is also called = {(a, a) : a  A} is called the identity relation on A.
image of ‘a’ under R. (d) Reflexive Relation : A relation R on a set A is said to
1.2.2 Domain and range of a relation : If R is a relation be reflexive if every element of A is related to itself.
from A to B, then the set of first elements in R is Thus, R is reflexive  (a, a)  R for all a  A.
called domain & the set of second elements in R is
(e) Symmetric Relation : A relation R on a set A is
called range of R. symbolically.
said to be a symmetric relation iff
Domain of R = { x : (x, y)  R}
(a, b)  R  (b, a)  R for all a, b  A
Range of R = { y : (x, y)  R}
i.e. aRb  bRa for all a, b  A.
The set B is called co-domain of relation R.
(f) Antisymmetric Relation : A relation R on set A is
Note that range  co-domain. said to be antisymmetric relation iff
(a, b)  R and (b, a)  R  a = b for all a, b  A
(g) Equivalence Relation : A relation R on a set A is
said to be an equivalence relation on A iff
(i) it is reflexive i.e. (a, a)  R for all a  A
Total number of relations that can be defined from a set
(ii) it is symmetric i.e. (a, b)  R  (b, a)  R for
A to a set B is the number of possible subsets of A × B. If
all a, b  A
n(A) = p and n(B) = q, then n(A × B) = pq and total number
pq
of relations is 2 . (iii) it is transitive i.e. (a, b)  R and (b, c)  R 
(a, c)  R for all a, b, c  A.
RELATION , FUNCTION & INVERSE TRIGONOMETRIC FUNCTIONS 3

(ii) If domain  R or R – {some finite points}


2. FUNCTIONS
Put y = f(x)
2.1 Definition Then express x in terms of y. From this find y for x to be
defined. (i.e., find the values of y for which x exists).
A relation ‘f’ from a set A to set B is said to be a function if (iii) If domain  a finite interval, find the least and greater
every element of set A has one and only one image in set B. value for range using monotonocity.
Notations

x f y (= f (x))
(Domain) input output (Range)
(independent variable) (dependent variable)
Two functions f & g are said to be equal iff
1. Domain of f = Domain of g
Domain
2. Co-Domain of f = Co-domain of g
f: A B co-domain. 3. f(x) = g(x)  x  Domain.

(read as : f is a function 2.3 Types of Functions


from set A to set B)
Definition 1 : A function f : X  Y is defined to be one-
2.2 Domain, Co-domain and Range of a function one (or injective), if the images of distinct
elements of X under f are distinct, i.e., for
Domain : When we define y = f (x) with a formula and the every x1, x2  X, f (x1) = f (x2) implies
domain is not stated explicitly, the domain is x1 = x2. Otherwise, f is called many-one.
assumed to be the largest set of x–values for which
Definition 2 : A function f : X  Y is said to be onto
the formula gives real y–values.
(or surjective, if every element of Y is the
The domain of y = f (x) is the set of all real x for image of some element of X under f, i.e.,
which f (x) is defined (real). for every y  Y, there exists an element x
Algo Check : Rules for finding Domain : in X such that f (x) = y.

(i) Expression under even root (i.e. square root, fourth


root etc.) should be non–negative. Range = Co-domain
f f
(ii) Denominator  0.
(iii) logax is defined when x > 0, a > 0 and a  1.
(iv) If domain of y = f (x) and y = g(x) are D1 and D2
respectively, then the domain of f (x) ± g(x) or f (x)
. g(x) is D1  D2. While domain of A B A B
(one-to-one) & (onto) (one-to-one) & (into)
f x
is D1  D2 – {x: g(x) = 0}.
gx
Range = Co-domain
Range : The set of all f -images of elements of A is known f f
as the range of f & denoted by f (A).
Range = f (A) = {f (x) : x  A};
f (A)B {Range Co-domain}.
Algo Check : Rule for finding range : A B A B
First of all find the domain of y = f (x) (many-to-one) & (onto) (many-to-one) & (into)

(i) If domain  finite number of points


range  set of corresponding f (x) values.
4 RELATION , FUNCTION & INVERSE TRIGONOMETRIC FUNCTIONS

Methods to check one-one mapping


2.5 Composite Functions
1. Theoretically : If f (x1) = f (x2)
 x1 = x2, then f (x) is one-one. Let us consider two functions,  f : X  Y1 and g : Y1  Y. We
define function h : x  y; such that
2. Graphically : A function is one-one, iff no line parallel
to x-axis meets the graph of function at more than h (x) = g (f (x)) = (gof ) (x).
one point.
3. By Calculus : For checking whether f (x) is One-One,
find whether function is only increasing or only
decreasing in their domain. If yes, then function is
one-one,

i.e. if f '  x   0,  x  domain

or if f '  x   0,  x  domain, To obtain h (x), we first take f–image of an element x  X so


that f (x)  Y1, which is the domain of g (x). Then take g–
then function is one-one. image of f (x), i.e., g ( f (x)) which would be an element of Y.
2.4 Even and Odd Functions

Even Function : f (–x) = f (x),  x  Domain


The graph of an even function y = f (x) is symmetric about
the y–axis. i.e., (x, y) lies on the graph  (–x, y) lies on
the graph. It should be noted that gof exists iff; the range of
f domain of g. Similarly, fog exists; iff; the range
Y–axis y = x2 of g   domain of f.

(–x, y) (x, y) 2.6 Inverse of Function

Definition : A function f : X  Y is defined to be invertible, if


X–axis there exists a function g : Y  X such that gof = IX
O
and fog = IY. The function g is called the inverse of
–1
Graph of an even function f  and is denoted by f  .
Let f : A  B be a one–one and onto function, then
Odd Function : f (– x) = –f (x),  x  Domain there exists a unique function, g : B  A such that
The graph of an odd function y = f (x) is symmetric about f (x) = y  g (y) = x,  x  A and y  B. Then g
origin i.e. if point (x, y) is on the graph of an odd function, is said to be inverse of f.
then (–x, –y) will also lie on the graph.
Thus, g  f
1
:BA  f  x  , x | x, f  x   f 
Let us consider one–one function with domain A and range B.
3
Y–axis y=x where A= {1, 2, 3, 4} and B = {2, 4, 6, 8} and   f  : A B is given by
(x, y) –1
f (x) = 2x, then write f and  f  as a set of ordered pairs.
Here, member y  B arises from one and only one member x  A.

X–axis
O

(–x, –y) Graph of an odd function


RELATION , FUNCTION & INVERSE TRIGONOMETRIC FUNCTIONS 5

So,    f = {(1, 2) (2, 4) (3, 6) (4, 8)} Properties of inverse of a function
–1
and  f  = {(2, 1) (4, 2) (6, 3) (8. 4)} (i) The inverse of bijection is unique.
(ii) The inverse of bijection is also bijection.
(iii) If  f : A  B is bijection and g : B  A is inverse of f,
then fog = IB and gof = IA.
Where, IA and IB are identity function on the sets A
and B respectively.
–1 (iv) If  f  : A  B and g : B  C are two bijections, then
In above function, Domain of f = {1, 2, 3, 4} = range of f –1 –1 –1
–1 gof : A  C is bijection and (gof ) = (f   og ).
Range of f  = {2, 4, 6, 8} = domain of f 
(v) In general, fog gof but if, (fog) (x) = (gof) (x) = x.
Which represents for a function to have its inverse, it –1 –1
then f  = g and g = f .
must be one–one onto or bijective.
3. BINARY OPERATIONS
Graph of the inverse of an invertible function :
Definition 1 : A binary operations * on a set A is a
Let (h, k) be a point on the graph of the function f. Then
function* : A × A  A. We denote * (a, b)
(k, h) is the corresponding point on the graph of inverse of
–1 by a * b.
f  i.e.,  f  .
Definition 2 : A binary operation * on a set X is called
commutative, if a * b = b * a, for every
a, b  X.
Definition 3 : A binary operations * : A × A  A is said
to be associative if (a * b) * c = a * (b * c),
 a, b, c,  A.
Definition 4 : Given a binary operation * : A × A  A,
an element e  A, if it exists, is called
identity for the operation *,
if a * e = a = e * a,  a  A.
The line segment joining the points (h, k) and (k, h) is
3.1 General Results
bisected at right angle by the line y = x.
So that the two points play object–image role in the line If x, y are independent variable then ;
y = x as plane mirror.
(i) f (xy) = f (x) + f (y)  f (x) = k ln x or f(x) = 0
It follows that the graph of y = f (x) and its inverse written
in form y = g (x) are symmetrical about the line y = x. (ii) f (xy) = f (x) . f (y)  f (x) = xn , n  R.
(iii) f (x + y) = f (x) . f (y)  f (x) = akx.
(iv) f (x) takes rational values for all x  f (x) is constant
function.
(v) By considering a general nth degree polynomial and
writting the expression.

1 1
f (x)  f    f (x)  f 
x x

 f (x )   xn  1  1  xn
6 RELATION , FUNCTION & INVERSE TRIGONOMETRIC FUNCTIONS

INVERSE TRIGONOMETRIC FUNCTIONS

INVERSE CIRCULAR FUNCTIONS

Function Domain Range

LM  ,  OP
1. y = sin-1 x iff x = sin y –1 < x < 1,
N 2 2Q
2. y = cos-1 x iff x = cos y –1 < x < 1 [0,  ]

FG   ,  IJ
3. y = tan-1 x iff x = tan y   x   H 2 2K
4. y = cot-1 x iff x = cot y   x   (0,  )

LM  .0IJ  FG 0,  OP
5. y = cosec-1 x iff x = cosec y  ,  1  [1, ] N 2 K H 2Q
LM0.  IJ  FG  , OP
6. y = sec-1 x iff x = sec y  ,  1  [1, ] N 2K H 2 Q

(i) Sin–1 x & tan–1 x are increasing functions in their domain.


(ii) Cos–1 x & cot–1 x are decreasing functions in their domain.

PROPERTY – I
{  x [–1, 1] and (/2–) [0, ])
–1 1
(i) sin x + cos x = /2, for all x [ –1, 1]  + cos–1 x = /2 ... (ii)
Proof. Let, sin–1 x =  ... (i) from (i) and (ii), we get
then, [– /2, /2] ( x [–1, 1])

 – /2 /2 sin–1 x + cos–1 x =
2
 – /2 – /2 (ii) tan–1 x + cot–1 x = /2, for all x R

 0

 
Proof. Let, tan–1 x =  ... (i)
2
then, (–/2, /2) {  x R)

 – [0, ]  
2   
2 2
Now, sin–1 x = 
 
 x = sin      
2 2

 
 x  cos     
2   0  
2

 cos1 x  
2
RELATION , FUNCTION & INVERSE TRIGONOMETRIC FUNCTIONS 7

  1
      (0, )   sin 
2  x

Now, tan–1 x =  1
{  x (–, –1] [1, )  [–1, 1] – {0}
 x = tan  x
 x = cot (/2 – ) cosec–1 x = [– /2, /2] – {0}

 1
 cot 1 x   {  /2 – (0, )}    sin 1   ... (ii)
2 x

 from (i) and (ii); we get


   cot 1 x  ... (ii)
2
1
from (i) and (ii), we get sin 1    cos ec1x
x
tan–1 x + cot–1 x = /2
1
 (ii) cos–1   = sec–1 x, for all x  (– , 1] [1, )
(iii) sec–1x + cosec–1 x = , for all x (– , –1] [1, ) x
2
Proof. Let, sec–1 x =  ... (i) Proof. Let, sec–1 x =  ... (i)

then, [0, ] – {/2} {  x  (–, –1] [1, )} then, x (–, 1] [1, ) and [0, ] – {/2}

 0 , /2 Now, sec–1 x = 

 – – 0, /2  x = sec 

    1
    ,    0   cos 
2 2 2 2 x

      1
   cos 1   ... (ii)
       ,  ,    0 x
 2   2 2 2
Now, sec–1 x =   x  (,  1]  (1, )

 x = sec  
 1
 x = cosec (/2 – )  x  [1, 1]  {0} and   [0, ]

 cosec–1 x = /2 – 
from (i) & (ii), we get
       
        ,  ,    0  1
cos1    sec 1 (x)
 2   2 2 2  x
 + cosec–1 x = /2 ... (ii)
1  cot 1 x , for x  0
from (i) and (ii); we get (iii) tan–1    
sec–1 x + cosec–1 x = /2  x    cot 1 x, for x  0

PROPERTY – II Proof. Let cot–1 x = . Then xR, x 0 and (0, ) ... (i)
Now two cases arises :
1 Case I : When x > 0
(i) sin–1   = cosec–1 x, for all x  (– , 1] [1, )
x In this case, (0, /2)
Proof. Let, cosec–1 x =  ... (i)  cot–1 x = 
then, x = cosec   x = cot 
8 RELATION , FUNCTION & INVERSE TRIGONOMETRIC FUNCTIONS

1 PROPERTY – III
  tan 
x
(i) cos–1 (–x) =  – cos–1 (x), for all x [–1, 1]
1 (ii) sec–1 (–x) = – sec–1 x, for all x (–, –1] [1, )
  tan 1   ... (ii)
x (iii) cot–1 (–x) = – cot–1 x, for all x R
from (i) and (ii), we get {  (0, /2)} (iv) sin–1 (–x) = – sin–1 (x), for all x [–1, 1]
(v) tan–1 (–x) = – tan–1 x, for all x R
1 (vi) cosec–1 (–x) = – cosec–1 x, for all x (–, –1] [1, )
tan 1    cot 1 x, for all x  0.
x
Proof. (i) Clearly, – x [–1, 1] for all x [–1, 1]
Case II : When x < 0 let cos–1 (–x) =  ... (i)
In this case (/2, ) {  x = cot < 0) then, – x = cos 
 x = – cos 

Now,      x = cos (– )
2
{  x (–1, 1) and – [0, ] for all [0, ]
 cos–1 x = – 
  0
2  = – cos–1 x ... (ii)
 – (–/2, 0) from (i) and (ii), we get
 cot–1 x =  cos–1 (–x) = – cos–1 x
 x = cot  Similarly, we can prove other results.
(iv) Clearly, – x [–1, 1] for all x [–1, 1]
1
  tan  let sin–1 (–x) = 
x
then, – x = sin  ... (i)
1  x = – sin 
   tan (  )
x  x = sin (–)
 – = sin–1 x
1
  tan (  ) {  tan (–) = – tan } {  x [–1, 1] and – [–/2, /2] for all [–/2, /2]
x
 = – sin–1 x ... (ii)
1 from (i) and (ii), we get
 – = tan–1   {  – (–/2, 0)}
x sin–1 (–x) = – sin–1 (x)

1 PROPERTY – IV
 tan 1       ... (iii)
x
(i) sin (sin–1 x) = x, for all x [–1, 1]
from (i) and (iii), we get (ii) cos (cos–1 x) = x, for all x [–1, 1]
(iii) tan (tan–1 x) = x, for all x R
1
tan       cot 1 x, if x  0
1
(iv) cosec (cosec–1 x) = x, for all x (–, –1] [1, )
x
(v) sec (sec–1 x) = x, for all x (–, –1] [1, )
Hence, (vi) cot (cot–1 x) = x, for all x R
1 Proof. We know that, if f : A  B is a bijection, then f–1 : B A
 1   cot x,
1 for x  0
tan     1
exists such that fof–1 (y) = f (f–1 (y)) = y for all y B.
 x    cot x, for x  0
Clearly, all these results are direct consequences of this
property.
RELATION , FUNCTION & INVERSE TRIGONOMETRIC FUNCTIONS 9

Aliter : Let   [–/2, /2] and x  [–1, 1] such that


sin = x.
Repeated Curve Main Curve Repeated Curve
then, = sin–1 x
x

y=
 x = sin = sin (sin–1 x) y= x or y = 0

-

 O  3
Hence, sin (sin–1 x) = x for all x [–1, 1]

-x
2 2 2

Similarly, we can prove other results.


Remark : It should be noted that,
sin–1 (sin ) , if [– /2, /2]. Infact, we have Thus, the graph for y = sin–1 (sin x), is a straight line up and
a straight line down with slopes 1 and –1 respectively lying
   , if   [3 / 2,   / 2]
 ,   
 if   [ / 2,  / 2] between   , .
sin 1 (sin )    2 2 
   , if   [ / 2, 3 / 2]
 2  , if   [3 / 2, 5 / 2] and so on.

Similarly,

Students are adviced to learn the definition of sin–1 (sin x) as,


 , if   [, 0]
 , if   [0, ]

cos 1 (cos )  
 2  , if   [, 2]  5 3
2  , if   [2, 3] and so on.  x  2 ;  x
2 2

   x 3 
;  x
 2 2
  , if   (3 / 2,   / 2) 
  
 ,
 if   ( / 2,  / 2) y  sin 1 (sin x)   x ;  x
tan 1 (tan )    2 2
   , if   ( / 2, 3 / 2)   3
   2, if   (3 / 2, 5 / 2) and so on.  x ; x
 2 2
 3 5
PROPERTY – V  x  2 ;
2
x
2
... and so on

(i) Sketch the graph for y = sin–1 (sin x)


(ii) Sketch the graph for y = cos–1 (cos x).
Proof. As, y = sin–1 (sin x) is periodic with period 2.
Proof. As, y = cos–1 (cos x) is periodic with period 2.
 to draw this graph we should draw the graph for one interval
of length 2and repeat for entire values of x.  to draw this graph we should draw the graph for one interval
of length 2and repear for entire values of x of length 2.
As we know,
As we know;
  
 x;  x
2 2  x; 0x
sin 1 (sin x)   cos 1 (cos x)  
 2  x; 0  2  x  ,
(  x);      x    i.e.,   x  3 
  
2 2 2 2 
 x; 0x
or cos 1 (cos x)  
 2  x;   x  2,
  
1
 x,  x
2 2 Thus, it has been defined for 0  x  2that has length 2.
or sin (sin x)  
  x,  3 So, its graph could be plotted as;
x ,
 2 2

which is defined for the interval of length 2 , plotted as ;


10 RELATION , FUNCTION & INVERSE TRIGONOMETRIC FUNCTIONS

y FORMULAE
xy
(i) tan–1 x + tan–1 y = tan–1 , xy < 1
–x x 1  xy
x or y = 0
O xy
(ii) tan–1 x – tan–1 y = tan–1 , xy > – 1
1  xy
Thus, the curve y = cos–1 (cos x).
2x
(iii) Sketch the graph for y = tan–1 (tan x). (iii) 2 tan–1 x = tan–1 ,|x|<1
1 x2
Proof. As y = tan–1 (tan x) is periodic with period .
 to draw this graph we should draw the graph for one interval 2x
(iv) 2 tan–1 x = sin–1 , | x | 1
of length and repeat for entire values of x. 1 x2

  
As we know; tan–1 (tan x) =  x;   x   1  x2
(v) 2 tan–1 x = cos–1 , x 0
 2 2 1 x2

  (vi) sin–1 x + sin–1 y = sin–1 (x 1  y 2  y 1  x 2 )


Thus, it has been defined for   x  that has length .
2 2
So, its graph could be plotted as; (vii) sin–1 x – sin–1 y = sin–1 (x 1  y 2  y 1  x 2 )

y
(viii) cos–1 x + cos–1 y = cos–1 (xy – 1  x 2 1  y2 )

x
(ix) cos–1 x – cos–1 y = cos–1 (xy + 1  x 2 1  y2 )
O

Thus, the curve for y = tan–1 (tan x), where y is not defined

for x(2n+1) .
2
11 RELATION , FUNCTION & INVERSE TRIGONOMETRIC FUNCTIONS

SOLVED EXAMPLES
(a, b)  R and (b, c)  R  f (a) = f (b) and f (b) = f (c)  f (a)
RELATION AND FUNCTION–II
= f (c)  (a, c)  R, which implies that R is transitive. Hence,
R is an equivalence relation.
Example – 1
Example – 4
Let L be the set of all lines in a plane and R be the relation
in L defined as R = {L1, L2} : L1 is perpendicular to L2}. Let A = {2, 3, 4, 5, 6, 7, 8, 9}. Let R be the relation on A
Show that R is symmetric but neither reflexive nor defined by
transitive. R = {(x, y) : x  A, y  A and x divides y}. Find
(i) R; (ii) domain of R; (iii) range of R; (iv) R–1
Sol. R is not reflexive, as a line L1 can not be perpendicular to
itself, i.e., (L1, L1)  R, R is symmetric as (L1, L2)  R. State whether or not R is (a) reflexive (b) symmetric
(c) transitive.
L3
Sol. Here, x R y iff x divides y, therefore,
L2
(i) R = {(2, 2), (2, 4), (2, 6), (2, 8), (3, 3), (3, 6), (3, 9), (4, 4),
(4, 8), (5, 5) (6, 6) (7, 7), (8, 8) (9, 9)}
L1
(ii) Domain of R = {2, 3, 4, 5, 6, 7, 8, 9} = A
 L1 is perpendicular to L2 (iii) Range of R = {2, 3, 4, 5, 6, 7, 8, 9} = A
–1
 L2 is perpendicular to L1 (iv) R = {(y, x) : (x, y)  R}
 (L2, L1)  R. = {(2, 2), (4, 2), (6, 2), (8, 2) (3, 3), (6, 3), (9, 3),
R is not transitive. Indeed, If L1 is perpendicular to L2 and L2 (4, 4), (8, 4) (5, 5) (6, 6), (7, 7) (8, 8), (9, 9)}
–1
is perpendicular to L3, then L1 can never be perpendicular to Infact R is {(y, x) : x, y  A, y is divisible by x}.
L3. In fact, L1 is parallel to L3, i.e., (L1, L2)  R, (L2, L3)  R but
(a) As (2, 2) (3, 3), (4, 4), (5, 5), (6, 6), (7, 7), (8, 8),
(L1, L3)  R.
(9, 9) belong to R, therefore, (R) reflexive.
Example – 2 (b) Here, R is not symmetric. We may observe that
(2, 4)  R but (4, 2)  R. Infact, ‘x divides y’ does not
Show that the relation R in the set Z of integers given by imply ‘y divides x’ when x  y.
R = {(a, b) : 2 divides a – b} (c) As x’ divides y’ and ‘y divides z’ imply ‘x divides z’,
is an equivalence relation. therefore, the relation R is transitive.

Example – 5
Sol. R is reflexive, as 2 divides (a – a) for all a  Z. Further, if
(a, b)  R, then 2 divides a – b. Therefore, 2 divides b – a. Determine which of the following binary operations on
Hence, (b, a)  R, which shows that R is symmetric. Similarly, the set R are associative and which are commutative.
If (a, b)  R and (b, c)  R, then a – b and b – c are divisible
by 2. Now, a – c = (a – b) + (b – c) is even (Why?). So (a – c) (a) a * b = 1  a, b  R
is divisible by 2. This shows that R is transitive. Thus. R is
an equivalence relation in Z.
a  b
(b) a * b =  a, b  R
2
Example – 3
Sol. (a) Clearly, by definition a * b = b * a = 1,
Let f : X  Y be a function. Define a relation R in X given by
R = {(a, b) : f (a) = f (b)}. Examine whether R is an equivalence  a, b  R. Also (a * b) * c = (1) * c = 1 and
relation or not. a * (b * c) = a * (1) = 1,  a, b c  R. Hence R is both
Sol. For every a  X, (a, a)  R, since f (a) = f (a), showing that R associative and commutative.
is reflexive. Similarly, (a, b)  R  f (a) = f (b)  ab ba
(b) a * b   = b * a, shows that * is
f (b) = f (a)  (b, a)  R. Therefore, R is symmetric. further, 2 2
12 RELATION , FUNCTION & INVERSE TRIGONOMETRIC FUNCTIONS

commutative. Further, Example – 7


ab
(a * b) * c =  *c
 2   1 x 2 
Find domain for f  x   sin –1  .
 2x 
ab
   c a  b  2c
= 2   .
2 4  1 x 2 
Sol. f  x   sin –1   is defined for ;
bc  2x 
But a * (b * c) = a *  
 2 
1 x 2 1 x 2
bc – 1 1 or 1
a 2x 2x
 2  2a  b  c  a  b  2c
2 4 4  |1 + x2|  |2x|, for all x
in general.  1 + x2  |2x|, for all x. {as 1 + x2 > 0}
Hence * is not associative.  x2 – 2|x| + 1  0
Example – 6  |x|2 – 2|x| + 1  0 {as x2 = |x|2}
 (|x| – 1)2  0
Find the domain of
But (|x| – 1)2 is either always positive or zero.
1
f x  . Thus, (|x| – 1)2 = 0
l og1/2  x – 7x 13
2
 |x| = 1
 x=±1
1 Thus, domain for f (x) is {–1, 1}
Sol. f x  . exists if;
l og1/2  x – 7x 13
2
Example – 8

         log1/2 (x2 – 7x + 13) > 0 Find the range of the function :


 (x2 – 7x + 13) < 1 ...(i)
and x2 – 7x + 13 > 0 ...(ii)
 
2
considering equation (ii), x – 7x + 13 > 0, we have f  x   3sin x + 8cos  x –   5
 3
 2 49  49
 x – 7x    13 – 0
 4  4
 
2
Sol. Here f  x   3sin x + 8cos  x –   5
 7 3  3
 x –    0
 2 4
= 3sin x + 4(cos x + 3 sin x) + 5
which is true for all x R
2
= (3 + 4 3 ) sin x + 4 cos x + 5.
 7
as  x –   0 for all x. ....(a)
 2 Put 3 + 4 3 = r cos  and 4 = r sin  so that
again taking (i), x2 – 7x + 13 < 1 4
+ + r  73  24 3 and  = tan–1
34 3
3 – 4
2
 x – 7x + 12 < 0  f (x) = 73  24 3 sin (x + ) + 5
 (x – 3) (x – 4) < 0
 3<x<4 ....(b)  Range of f (x) is
combining (a) and (b), we have 5 – 73  24 3 , 5  73  24 3  .
Hence domain of f (x)  (3, 4) or ]3, 4[  
RELATION , FUNCTION & INVERSE TRIGONOMETRIC FUNCTIONS 13

Example – 9 the least value of

The range of the function sin2 x – 5 sin x – 6 is  2 2 


f (x) = tan  , when x  .
9 9 3
2
Sol. Here, f (x) = sin x – 5 sin x – 6
 The greatest value of f  x   3 and the least value
 2 25  25
=  sin x  5sin x    6  of f (x) = 0.
 4  4
 Range of f  0, 3 
2

 5  49
=  sin x    ...(i)
 2 4 Example – 12

2 Find the period of the function.


9  5 49
where   sin x    ...(ii) f (x) = sin x + {x}
4  2 4

 From Eqs (i) and (ii), Sol. Here f (x) = sin x + {x}
– 10  f (x)  0 Period of sin x is 2 and that of {x} is 1.
 Range of f (x)  [–10, 0]. But the L.C.M. of 2 and 1 does not exist.
Example – 10 Hence sin x + {x} is not periodic.

Example – 13
Find the range of the function :
Find the period of the function
f  x   ln x 2  4x  5
x
f (x) = tan + sin 2x
3
2
Sol. Here  f (x)  l n x 2  4x  5  ln  x  2 1
Sol. Here f (x) = tan x/3 + sin 2x.
i.e. x2 + 4x + 5 takes all values in [1, )
Here tan (x/3) is periodic with period 3 and sin 2x is periodic
 f (x) will take all values in [0, ).
with period .
Hence range of f (x) is [0, ).
Hence f (x) will be periodic with period 3.
Example – 11
Example – 14
Find the range of the function
Find the period of the function
2 f (x) = |sin x| + |cos x|.
f  x   tan  x 2 is
9
Sol. Here f (x) = |sin x| + |cos x|

2 Now, |sin x| = sin 2 x =


1– cos 2x
, which is periodic
Sol. For f (x) to be defined,  x2  0
9 2
with period .
 
  x Similarly, |cos x| is periodic with period .
3 3
Hence, according to rule of LCM, period of f (x) must
   be .
 Domain of f    ,  .
 3 3
   
But sin   x  = cos x and cos   x  = sin x
2  2 
2
The greatest value of f (x) = tan  0 , when x = 0 and
9 Since /2 < , period of f (x) is /2.
14 RELATION , FUNCTION & INVERSE TRIGONOMETRIC FUNCTIONS

Example – 15  IA (x) = IA (y)


 x = y.
Let f : R R be defined by f (x) = 3x – 2 and g : R  R be
So, f (x) = f (y)  x = y  f is one-one. Thus, we see that f is
x2 both one-one and onto i.e. f is a bijection. Similarly, we can
defined by g  x   . Show that fog = IR = gof. show that g is a bijection.
3
Moreoever, for all x  A,
Sol. For all x  R, (fog) (x) = f (g (x))
x = IA(x) = (fog) (x) = f (g (x))
–1
x2  x2  x = f (g(x))  f (x) = g (x)
=f    3  – 2 = x = IR (x)
 3   3  
–1
f = g.
Hence fog = IR Example – 18
Again, (gof) (x) = g (f (x))

 3x  2   2 ex – e – x
= g (3x – 2) = = x = IR (x) Let f : R  R be defined by f  x   . Is f  (x)
3 2
 gof = IR. invertible ? If so, find its inverse.

Example – 16 Sol. Let us check invertibility of f (x) :

Let X = {–2, –1, 0, 1, 2, 3} and Y = {0, 1, 2, ..., 10} and ex  e– x


(a) One-one : Here, f   x  
f : X  Y be a function defined by f (x) = x2 for all 2
x  X, find f –1 (A) where (A) = {0, 1, 2, 4}.
e 2x  1
 f  x   which is strictly increasing as
–1 –1
Sol. Here, we have to find f (0), f (1), f (2) and f (4).
–1 –1 2e x
2 –1 e2x > 0 for all x.
Now f (x) = 0  x = 0  x = 0  f (0) = {0},
2 Thus, one-one.
f (x) = 1  x = 1  x = –1, 1
–1 2 (b) Onto : Let y = f (x)
 f (1) = {–1, 1}, f (x) = 2  x = 2
ex – e – x
 x=– 2, 2 but none of these is in X.  y where y is strictly monotonic.
2
–1 2
 f (2) = , f (x) = 4  x = 4  x = – 2, 2 Hence, range of f (x) = ( f (–)), f ())
–1
 f (4) = {–2, 2}.  range of f (x) = (–, )
–1
Hence, f (A) = {0, –1, 1, –2, 2}. So range of f (x) = co-domain.
Example – 17 Hence, f (x) is one-one and onto.

e 2x – 1
Let A be a non-empty set and f : A  A, g : A  A (c) To find f–1 : y 
2e x
be two functions such that fog = I A = gof, show
that f and g are bijections and that g = f  –1 .  e2x – 2ex y – 1 = 0

2y  4y 2  4
Sol. Consider f : A  A, Let y  A be arbitrary. Since fog = IA,  ex 
2
therefore, (fog) (y) = y
 f (g (y)) = y  x  l og y  y 2 1 
 f (t) = y, where t = g (y)  A.
This means that for y  A, there exists t  A s.t. f (t) = y.
 
f –1  y   l og y  y 2 1 
Hence f is onto. [as f (x) = y  x = f –1(y)]
Let x, y  A s.t. f (x) = f (y) f –1
x
Since, e is always positive.
 g (f (x)) = g (f (y))
So, neglecting negative sign.
( g is a function)
 (gof ) (x) = (gof ) (y)
–1 2
Hence, f  x   l og x + x 1  
RELATION , FUNCTION & INVERSE TRIGONOMETRIC FUNCTIONS 15

Example – 19 Example – 20

Let f : [1/2, )  [3/4, ), where f (x) = x2 – x + 1. If f (x) = x2 – 3x + 2 be a real valued function of the real
Find the inverse of f (x). Hence, solve the equation variable, find fof.

1 3
x2 – x + 1 =  x . Sol. We are given the function f : R  R, defined by
2 4 2
f (x) = x – 3x + 2 for all x  R.
2
Now, ( fof ) (x) = f (f (x)) = ( f (x)) – 3 f (x) + 2
2
Sol. (a) f (x) = x – x + 1 2 2 2
= (x – 3x + 2) – 3 (x – 3x + 2) + 2
4 2 3 2 2
2 = x + 9x + 4 – 6x – 12x + 4x – 3x + 9x – 6 + 2
 1 3
 f (x) =  x    , which is clearly one-one and 4 3
= x – 6x + 10x – 3x.
2

 2 4
Example – 21
onto in given domain and co-domain.
(b) Thus, its inverse can be obtained. Two functions are defined as under :

Let f (x) = y  x  1, x 1
f x   ,
2x  1, 1 x  2
2
 1 3
 y  x   
 2 4  x 2 , – 1 x  2
g x  
x  2, 2  x  3

1 3 Find fog and gof.


 x   y
2 4

 g  x  1, g  x  1
Sol. ( fog) (x) = f (g(x))  
x
1
 y
3 –1  2g  x  1, 1 g  x   2
 [f (x) = y  x = f (y)]
2 4
Let us consider, g(x)  1 :
(i) x2  1, –1  x < 2
1 3
 f 1  y    y  –1  x  1, –1  x  2
2 4
 –1  x  1
[neglecting –ve sign as always +ve] (ii) x + 2  1, 2  x < 3
 x  –1, 2  x  3
1 1 3  x = .
 f x   x 
2 4
Let us consider, 1 < g(x)  2,
(iii) 1 < x2  2, –1  x < 2
2 1 3
(c) To solve x –x+1=  x  , as f (x) = f –1 (x) has
2 4   
x   – 2 ,– 1  1, 2  , –1  x < 2

only one solution in this case. 2.


 1  x 
ie, f (x) = x (iv) 1 < x + 2  2, 2  x  3
2
 x –x+1=x  –1 < x  0, 2  x  3, x = 
2
 x – 2x + 1 = 0 2
 x 1, –1 x 1
 (x – 1) = 0
2
Thus f  g  x     2
 2x 1, 1 x  2
x = 1 is the required solution.
Now, Let us consider gof :
16 RELATION , FUNCTION & INVERSE TRIGONOMETRIC FUNCTIONS

Example – 23
2
 f  x  , –1 f  x   2
gof = g  f  x    
 f  x   2, 2  f  x   3 Find whether the given function is even or odd ?

Let us consider –1 <  f (x) < 2 : x x


f x   1
x
e –1 2
(i) –1  x + 1 < 2, x  1
 –2  x  1, x  1
 –2  x  1 Sol. We have

(ii) –1  2x + 1 < 2, 1 < x  2


–x x –ex .x x
 –1  x  1/2, 1 < x  2 f  –x   – 1 
–x – 1
e –1 2 1 – ex 2
 x = .
Let us consider 2 <  f (x) < 3 :
(iii) 2  x + 1 < 3, x  1 
e x
– 1  1 x

x
1
e x
– 1 2
 1  x  2, x  1
 x=1
2  2x + 1 < 3, 1 < x  2 x x x x
(iv) x – 1  x
x
  1  f x
e –1 2 e –1 2
 1  2x  2, 1 < x  2
 1/2  x  1, 1 < x  2 Hence f (x) is an even function.
 x = .
Example – 24

 x 1 2 , –2  x 1 If f is an even function, find the real values of x satisfying


g  f  x   
 x  3, x 1
 x 1 
the equation f  x   f  .
2
If we like we can also write g ( f (x)) = (x + 1) , –2  x  1.  x2

Example – 22
Sol. Since, f (x) is even, so f (–x) = f (x)
Let f : RR defined by f (x) = x3 + ax2 + 3x + 100. Then find
the values of a for which f is a one-one function. x 1 x 1
Thus, x or x 
x2 x2
Sol. f (x) = x3 + ax2 + 3x + 100
2 2
 f (x) = 3x2 + 2ax + 3.  x + 2x = x + 1 or – x – 2x = x + 1
2 2
For f (x) to be one-one.  f(x) > 0 or < 0  x + x – 1 = 0 or – x – 3x –1 = 0

But f  (x) is a quadratic expression and coefficient of


1  5 3  5
x2 > 0 so that f(x) > 0  x or x
2 2
 D<0
 4a2 – 36 < 0
 1  5 1  5 3  5 3  5 
 a2 < 9 Thus, x   2 , 2
,
2
,
2 


 –3 < a < 3. Example – 23
RELATION , FUNCTION & INVERSE TRIGONOMETRIC FUNCTIONS 17

Example – 25
 997   999 
 f  f   1
 1996   1996 
9x
Let f (x) = x . Show f (x) + f (1 – x) = 1, and hence
9 3  998   998 
 f  f   1
evaluate  1996   1996 

 1   2   3   1995   998  1
f  f  f  ...  f . f
 1996   1996   1996   1996  or 
 1996  2
Adding all the above expressions, we get
x
9
Sol. f (x) = ...(i)  1   2   1995 
9x  3 f  f   ...  f 
 1996   1996   1996 
91 x
and f (1 – x) = 1 x 1
9 3 = (1 + 1 + 1 + ... + 997) +
2

9 1
9x  9 = 997 +
 f (1 – x) = 2
9
3 9  3.9 x
9 x = 997.5.

INVERSE TRIGONOMETRIC FUNCTIONS


9
f (1 – x) = 3  3  9x  ...(ii)
Example – 26

Adding Eqs. (i) and (ii), we get


 3 
x Prove that (i) sin–1      3 ,
9 9  2 
f (x) + f (1 – x) = 9 x  3  3 3  9 x
 
  5   
(ii) cos–1  cos     .
x
3.9  9 3  9  3
x
  3  3
= 
3  9  3
x
3  9 x  3
 3   
Sol. (i) Let sin–1      so that     ,  and
 f (x) + f (1 – x) = 1 ...(iii)  2   2 2

1 2 3 998 3
Now, putting x  , , ,...,  = sin 
1996 1996 1996 1996 2
in (Eq. (iii), we get     
 
 sin = – sin = sin    note that  3    2 ,
 1   1995 
3  3  2 
f  f   1
 1996   1996  
 3 
 = – sin–1      .
3  2  3
 2   1994 
 f  f   1
 1996   1996    5      
(ii) cos–1  cos    = cos–1  cos  2   
  3    3 
 3   1993 
 f  f   1
 1996   1996      
= cos–1  cos     ,
.........................................   3  3
.........................................

note that [0, ] = range of cos–1 x.
3
18 RELATION , FUNCTION & INVERSE TRIGONOMETRIC FUNCTIONS

Example – 27
  3   3
(iii) sin  2 cos 1     = sin (2 ), where = cos–1  
Evaluate   5   5

= 2 sin cos 
  3   4 
(i) sin  2sin 1    (ii) sin  2sin 1    
  5    5   1  3  
 3 
= 2 sin  cos 1     cos  cos    
  5    5 
  3    2 
(iii) sin  2cos 1     (iv) sin  3sin 1   
  5    5  2
 3  3
= 2 1        ( sin (cos–1 x) = 1  x 2 for | x |  1)
 5  5
  3  3
Sol. (i) sin  2 sin 1    = sin 2 , where = sin–1  
  5  5  4  3 24
 2       .
 5  5 25
= 2 sin cos 
  2  2
 3   1  3   (iv) sin  3 sin 1    = sin 3, where = sin–1  
= 2 sin  sin 1    cos  sin      5  5
 5
    5 
= 3 sin – 4 sin3 
2
3 3 3
= 2 1    (  cos (sin–1 x) = 1  x 2 for | x |  1) 2 2
5 5  3 4 
5 5

3 4 24
= 2    1  2  2
5 5 25    sin   ,  sin   
 5 5

  4  1 4  
(ii) sin  2 sin 1     sin  2 sin  6 32 118
  5   5     .
5 125 125
(  sin–1 (–x) = – sin–1 x)
Example – 28
= – sin  2 sin 1  4   Prove that
  5 
3  8   84 
(i) sin–1   – sin–1   = cos–1  85 
 4  5  17   
= – sin  2 sin 1   
  5 
3  8  77 
(ii) sin–1   + sin–1   = sin–1  
5  17   85 
4
= – sin 2 , where = sin–1   = 2 sin cos 
5
Sol. (i) Since sin–1 x is an increasing function in [–1, 1] and

 4    4  3 8
= – 2 sin  sin 1    cos  sin 1     , therefore,
5 17
  5    5 

3 8 3 8


4 4
2 sin–1   > sin–1   sin–1   – sin–1   > 0.
= 2   1    5  17  5  17 
5 5
3 8
 sin–1   – sin–1   [0, ] = range of cos–1 x
(  cos (sin–1 x) = 1  x 2 ) 5
   17 

8 3 24
  
5 5 25
RELATION , FUNCTION & INVERSE TRIGONOMETRIC FUNCTIONS 19

Example – 29
 1  3   1  8  
 0  sin    and 0  sin    
  5  2  17  2
4 5  16  
Show that sin–1   + sin–1   + sin–1    .
5  13   65  2
 3  8 
Now cos sin 1    sin 1   
 5  17  

4 5
  3   1  8    1  3   Sol. Let = sin–1   + sin–1  
= cos  sin 1    cos  sin    + sin  sin    sin 5  13 
  5    17     5 
then 0 < < 
 1  8  
 sin   
  17    1  4   1  5  
 0  sin    , 0  sin    
  5  2  13  2
9 64 3 8 4 15 3 8 84
 1 1       
25 289 5 17 5 17 5 17 85
 4 5
Now cos = cos  sin 1  sin 1 
 5 13 
3 8  84 
 sin 1    sin 1    cos 1   .
5  17   85 
 4  1 5   1 4 
(ii) We know that = cos  sin 1  cos  sin  – sin  sin 5  sin
 5  13 
3  8  3
0 sin–1  and 0  sin–1  , therefore, 0  sin-1   +  1 5 
5 2 17 2 5  sin 
 13 
8
sin–1    (  cos (A + B) = cos A cos B – sin A sin B)
 17 
We compute 2 2
4 5 4 5
 1   1    .
 3  8  5  13  5 13
cos  sin 1    sin 1   
  5   17  
( cos (sin 1 x)  1  x 2 )
 1  3    1  8  
= cos  sin    cos  sin   
  5    17   3 12 20 36  20 16  16 
 .      cos 1  
5 13 65 65 65  65 
 1  3    1  8  
– sin  sin    sin  sin   
  5    17   4 5  16 
Hence sin–1   + sin–1   + sin
–1
 
5  13   65 
2 2
3  8 3 8 4 15 3 8 36
 1   1           1  4  1  5   1  16 
5  17  5 17 5 17 5 17 85 = sin    sin     sin  
  5   13    65 

8  36   2 
3  36 
 sin–1   + sin–1   = cos–1   = sin–1  1      16 
= cos–1   + sin–1
 16 
5  17   85    85    
   65   65 

(  cos–1 x = sin–1 1  x 2 for 0 x  1) 


(  sin–1 t + cos–1 t = for – 1  t  1)
2
3 8  77 
 sin–1   + sin–1   = sin–1   , as desired. 
5  17   85  =
2
20 RELATION , FUNCTION & INVERSE TRIGONOMETRIC FUNCTIONS

Example – 30 Example – 32

Prove that cot–1 (13) + cot–1 (21) + cot–1 (–8) = . 


Find x if sin–1 x + sin–1 2x = .
3
Sol. L.H.S. = cot–1 (13) + cot–1 (21) + cot–1 (–8)

1  1  
= tan–1   + tan–1   +  – cot–1 8 Sol. Given sin–1 x + sin–1 2x = sin–1 2x = – sin–1 x
 13   21  3 3

(  cot–1 (–x) = – cot–1 x, x R)  1 


 2x = sin   sin x 
 3 
 1 1 
 13  21  1
= tan–1   + – tan–1    
8
 1  1 . 1   2x = sin cos (sin–1 x) – cos sin (sin–1 x)
 13 21  3 3

3 1 x 3
 2x  1  x2  x  2x   1 x2
1 2 2 2 2
 21  13 
= tan–1   + – tan
–1
 
 13  21  1  8 On squaring the two sides, we get

25 x 2 3 3 3
 34  1  (1  x 2 ) 28x2 = 3 x2 = 28  x  .
= tan–1  –1
 + – tan  8  4 4 28
 272 
= = R.H.S. 3
( x = – does not satisfy the given equation)
28
Example – 31
Example – 33
  4
Prove that 2 tan–1 (–3) = – + tan–1    .
2  3 x y
If cos–1   + cos–1   = ,
a b

1
Sol. L.H.S. = 2 tan–1 (–3) = – 2 tan–1 3 = – 2 tan–1 3 = – 2 cot–1   x 2 2xy y2
3 Prove that  cos   = sin2 .
a 2 ab b2

 1  1   1  1 
= 2   tan       2 tan   x y
 2 3
  3 Sol. Given cos–1   + cos–1   = 
a b

 2 (1/ 3) 
= – + tan–1  2  y x
1  (1/ 3)   cos–1   =  – cos–1  
b a

 2x 
 2 tan x  tan
1 1
for | x | 1 y   x 
 1  x 2
   cos    cos 1   
b   a 

3 4  4
= – + tan–1 = – + cot–1 = – + – tan–1   y x  1  x  
4 3 2 3   cos    + sin  sin  cos   
b a   a 

 4   4
=  tan 1      tan 1     R.H.S. 2
2 3
  2  3 y x x
  cos = sin  1   
b a a
RELATION , FUNCTION & INVERSE TRIGONOMETRIC FUNCTIONS 21

(  sin (cos–1 x) = 1  x 2 for |x| 1) Example – 36

2  x2  1
y x  1 3 cos–1 x = cos–1 (4x3 – 3x),  x  1.
  cos   = sin   a 2 
2
 2
b a   

Sol. Let cos–1 x = so that


y2 x 2  yx  x2 
  cos2
 – 2     cos = sin2
 1  2  x = cos and 0 
b2 a 2 ba   a 

1  1
x2 2 2
2 xy y2 As x   , 1 , therefore,  x  1
 (cos + sin ) – cos + = sin2   2  2
a2 ab b2

x 2 2xy y2 
  cos   2  sin 2   cos  cos  cos 0
a 2
ab b 3

Example – 34 
   0
3
If cos–1 x + cos–1 y + cos–1 z = ,
Prove that x2 + y2 + z2 + 2 xyz = 1. Note (that cos is decreasing in [0, ])

 3  0, i.e. 3 [0, ]


Sol. Let cos–1 x = A, cos–1 y = B, cos–1 z = C
so that x = cos A, y = cos B, z = cos C and A + B + C = .  cos–1 (4 x3 – 3x)

 x2 + y2 + z2 + 2xyz = cos2 A + cos2 B + cos2 C + 2 cos A cos B = cos–1 (4 cos3 – 3 cos )
cos C
= cos–1 (cos 3 )
1  cos 2A 1  cos 2B 1  cos 2C = 3  (Note that 0 3 )
=   + 2 cos A cos B cos C
2 2 2
= 3 cos–1 x.
3 1
  (cos 2A + cos 2B + cos 2 C) + 2 cos A cos B cos C Example – 37
2 2

3 1  x  1  x 
  (–1 –4 cos A cos B cos C) + 2 cos A cos B cos C Prove that tan–1    sin   , | x | < a.
2 2  2 2 a
 a x 
(Using a result from conditional identities)
= 1, as required.
x  
Example – 35 Sol. Let sin–1   =  x = a sin , and –   
a 2 2

2 sin–1 x = cos–1 (1–2x2), 0 x  1. (  | x | < a)

Now a 2  x 2  a 2  a 2 sin 2   a 2 cos 2  = a cos 


 
Sol. Let sin–1 x =  x = sin , –   
2 2    
      cos   0 
 2 2 

But 0  x  1 0   
2
 x  1  a sin  
Hence cos–1 (1–2x2) = cos–1 (1–2 sin2 ), 0 2 Hence tan 1    tan  
 2
= cos–1 (cos 2), 0 2   a x
2
  a cos  
=2
x
 cos–1 (1 – 2x2) = 2= 2 sin–1 x. = tan–1 (tan ) = = sin–1 .
a
22 RELATION , FUNCTION & INVERSE TRIGONOMETRIC FUNCTIONS

Example – 38
2
 2 tan  
Thus L.H.S.  (sin 2) 2   2 
 1  tan  
 1  x2  1   1
Show that tan–1     tan–1 x, x > 0
 x  2 2
  2
 
 2 1  x / 1  x 
 
  1   1  x  
Sol. Let = tan–1 x x = tan , 0      1  x  
2

 1 x2 1   1  tan 2   1   sec   1 
 tan 1    tan 1    tan 1  
4 (1  x) (1  x)
 x   tan    tan    = 1 – x2 = R.H.S.
    (1  x  1  x) 2

 1  Example – 40
 cos   1   1  cos  
= tan–1   = tan–1  =
 sin    sin    1  x2  1 x2   1
 cos      cos 1 (x 2 ) .
  Prove that tan–1 
 1 x2  1 x2  4 2
 

 2  
 2 cos 2 
 
tan–1  

 Sol. Let x2 = cos 2 so that 0 2 , i.e., 0   .
 2 sin cos  2 4
 2 2
Now, 1  x 2  1  cos 2  2 cos 2   2 cos 
      
= tan–1  cot  = tan–1  tan    
 2   2 2  and 1  x 2  1  cos 2  2 sin 2   2 sin 

   1
    tan 1 x .  1 x2  1 x2 
2 2 2 2  2 cos   2 sin  
Hence, tan–1  2 2
 = tan–1 
  2 cos   2 sin  
 1 x  1 x   
Example – 39

 1 x   2 cos   2 sin  
Prove that sin2  2 tan 1 2
 = 1 – x where – 1  x < 1.  
 1 x 2 cos   1  tan  
  –1   –1
= tan  = tan  
2 cos   2 sin    1  tan  
 
 2 cos  
  x 
Sol. L.H.S. sin2  2 tan 1 2
 = sin (2 ),
 1  x
 
     1
= tan–1  tan           cos 1 (x 2 )
  4   4 4 2

where  = tan–1 1 x
1 x 1
[  cos 2 = x2 2= cos–1 (x2) = cos–1 (x2)]
2

1 x
ie. tan =
1 x
RELATION , FUNCTION & INVERSE TRIGONOMETRIC FUNCTIONS 23

Example – 41
1 1 x2
  1, 0, 1   1, 0,1
tan (2 tan 1 x) 2x
 x 1  –1  x  1  
Solve the equation tan–1   + tan  x  2   4 ,
 x2   1 x2 1 x2 1  x2
  1,  0 or 1
|x| < 1. 2x 2x 2x
 x2 – 2x – 1 = 0, x2 = 1 or x2 + 2x – 1 = 0
 x 1  –1 
x 1    x=1± 2 , ± 1 or – 1 ± 2 .
Sol. Given equation is tan–1   + tan  
 x2  x2 4
Example – 43

 x 1 x  1 
      
tan 1  x2 x2 Solve : 2 tan–1 (cos x) = tan–1 (2 cosec x), x    ,
 
 2 2 
  x 1   x 1   4
1  x  2   x  2  
   
Sol. Given that,
2 tan–1 (cos x) = tan–1 (2 cosec x)
 1  x  y  
 tan x  tan y  tan 
1 1
 for xy  1  2 cos x 
  1  xy   tan–1  –1
  2  = tan (2 cosec x)
2  1  cos x 
 and for | x |  1,  x  1   x  1   1  x  1 
    2 
  x 2 x2 4x 
2 cos x
 = 2 cosec x
1  cos 2 x
(x  1) (x  2)  (x  1) (x  2) 
  tan
(x  2) (x  2)  (x  1) (x  1) 4 2 cos x 2
 
sin 2 x sin x
2x 2  4
  1 2x2 – 4 = – 3 cos x
(x  4)  (x 2  1)
2
 1 (  sin x  0)
sin x
1 1
 2x2 = 1 x2 = x = ± .    
2 2 cot x = 1 x = as x    , 
4  2 2
Example – 42
Example – 44
Solve the equation sin {2 cos–1 (cot (2 tan–1 x))} = 0.

Sol. Given equation is sin {2 cos–1 (cot (2 tan–1 x))} = 0 Solve for x : sin–1 (1 – x) – 2 sin–1 x = .
2
 2 cos–1 (cot (2 tan–1 x) = n , nI

n 
 cos–1 {cot (2 tan–1 x)} = , nI Sol. We have sin–1 (1–x) – 2 sin–1 x =
2 2

 
 cos–1 {cot (2 tan–1 x)} = 0, , sin–1 (1–x) = + 2 sin–1 x
2 2

(  cos–1 x lies in [0, ]) 1 – x = sin    2 sin 1 x 


2 

 cot (2 tan–1 x) = cos 0, cos , cos  1 – x = cos (2 sin–1 x)
2
1 – x = 1 – 2 sin2 (sin–1 x)
1 – x = 1 – 2 [sin (sin–1 x)]2
1 – x = 1 – 2 x2
24 RELATION , FUNCTION & INVERSE TRIGONOMETRIC FUNCTIONS

2x2 – x = 0 Example – 46
x (2x –1) = 0 Express each of the following in the simplest form :
1  1  cos x 
x = 0, (i) tan–1  , –  < x < 
2
 1  cos x 

1
For x = , sin–1 (1–x) – 2 sin–1 x  cos x   
2 (ii) tan–1  ,   x 
 1  sin x  2 2

 1 1
= sin–1  1   – 2 sin–1 Sol. (i) We have,
 2 2
 1  cos x 
tan 1  
1 1  1  cos x 
= sin–1 – 2 sin–1
2 2
 x 
2 sin 2 
 2   tan 1  tan 2 x   tan 1  tan x 
1   tan 1      
= – sin–1 =  R.H.S.  2 cos x 
2  2   2
2 6  2 

1  1  x
x= is not a solution of given equation. Hence, x = 0 is the tan   tan 2  , if    x  0
2   

 tan 1  tan x  , if 0  x  
required solution.  
2

Example – 45
 1    x   x
 tan tan      , if    x  0
–1 –1 –1    2  2
If tan x + tan y + tan z = , prove that x + y + z = xyz. 
 1  x x
tan tan   , if 0  x  
  2 2
Sol. Let tan–1 x = , tan–1 y = and tan–1 z = 
(ii) We have,
x = tan , y = tan and z = tan 
 x x 
Now, given that,  cos x   cos 2  sin 2 
tan  1 1 2 2
  tan  
tan–1 x + tan–1 y + tan–1 z =   1  sin x   cos 2 x  sin 2 x  2 sin x cos x 
 2 2 2 2
+ + = 

+ = –   x x x x   x x


  cos  sin   cos  sin    cos 2  sin 2 
1  2 2 2 2  1
tan (+ ) = tan (– )  tan  2   tan  
  x x    cos x  sin x 
  cos  sin    2 2
 2 2
tan   tan 
   tan 
1  tan  tan 

 x
Cross multiply, we have 1
1  tan 2   tan 1  tan    x  
 tan     
1  tan x   4 2 
tan + tan = – tan + tan tan tan 
 2
tan + tan + tan = tan tan tan 
 x
x + y + z = xyz. Hence, the result.  
4 2
RELATION , FUNCTION & INVERSE TRIGONOMETRIC FUNCTIONS 25

(ii) Putting x = a cos , we have


    x   x 
  2  x  2   4   2  4  0  4  2  2
  ax
tan 1
ALITER We have, ax

    a  a cos 
 sin  2  x    tan 1
 cos x 
1 1   a  a cos 
tan    tan  
 1  sin x  
1  cos   x  
 2   1  cos 
 tan 1
1  cos 
  x   x 
 2 sin  4  2  cos  4  2  
 tan 1      tan 1 cot    x   
     2sin 2
  x    4 2  2  tan 1  tan  
2 sin 2     tan 1
 
 4 2    2
2 cos 2
2
     x      x   x
 tan 1  tan        tan 1  tan       
 2  4 2    4 2  4 2 
   tan 1  tan 
 2
Example – 47
  
Write the following functions in the simplest form :  a  x  a  0      2  2 
 
 x 
(i) tan–1   , –a < x < a  1 x
 a  x 2
2
   cos 1
2 2 a
 a  x 
(ii) tan–1   , –a < x < a  x 1 x 
 a  x   x  a cos   cos   a    cos a 
 
 x  (iii) Putting x = a tan , we have
(iii) sin–1  
2 2
 x  a 
 x 
sin 1  
 x  a 2
2


Sol. (i) Putting x = a sin , we have


 a tan  
 sin 1  
 x  2 2
 a tan   a
2

tan 1  
2 2
 a  x 
 a tan  
   sin 1  
 tan 1
a sin   a sec  
 2 2 2

 a  a sin   = sin–1 (sin )

 a sin   x
1 x = = tan–1
 tan 1  1
  tan (tan )    sin a
 a cos   a

x  x 1 x 
 x  a tan   tan   a    tan a 

 x  a sin   sin   a   
 
 x 
   sin 1
 a 
26 RELATION , FUNCTION & INVERSE TRIGONOMETRIC FUNCTIONS

Example – 48  
 
4 2
Prove that :

 1 1 1  2
2 2
sin (2x 1  x ) ,if  x
 2 2

 1  –  – 2  –
2 sin 1 x     sin 1 (2x 1  x 2 ) , if  x 1 2
 2
 2 1 
   sin (2x 1  x ) , if  1  x    0 – 2
 2 2
1
Also,  x  1  0  2x 1  x 2  1
–1
Sol. Let sin x = . Then, 2
x = sin ,
 sin 2 = 2x 1  x 2 from (i)
 cos = 1  x 2
[  cos  > 0 for [–/2, /2]  sin (– 2) = 2x 1  x 2
 sin 2 = 2 sin cos  ... (i)  – 2= sin–1 (2x 1  x 2 )
 sin 2 = 2x 1  x 2
 – 2 sin–1 x = sin–1 (2x 1  x 2 )
1 1
CASE I When – x  2 sin–1 x = – sin–1 (2x 1  x 2 ).
2 2
We have, 1
CASE III When – 1 x < –
2
1 1
 x We have,
2 2
1
  –1x<–
   2
4 4
  1
   2   – 1 sin –
2 2 2

1 1  
Also,  x   
2 2 2 4

 –1 2x 1  x 2  1  – 2 –
2
 sin 2 = 2x 1  x 2 [From (i)] 
 0 + 2 
 2= sin–1 (2x 1  x 2 ) 2
1
 2 sin–1 x = sin–1 (2x 1  x 2 ) Also,–1  x  – –1  2x 1  x 2  0
2
1
CASE II When x1:  sin 2 = 2x 1  x 2 [From (i)]
2
We have,  – sin (+ 2) = 2x 1  x 2

1  sin (––2 ) = 2x 1  x 2
x1
2
 – – 2 = sin–1 (2x 1  x 2 )
1
  sin  1  2 = – – sin–1 (2x 1  x 2 )
2
 2 sin–1 x = –– sin–1 (2x 1  x 2 )
RELATION , FUNCTION & INVERSE TRIGONOMETRIC FUNCTIONS 27

Example – 49 Example – 50

Prove that : Prove that :

 1  cos x  1  cos x   x 3
 1  cos x  1  cos x   x tan 1     , if   x 
 1  cos x  1  cos x 4 2 2
tan 1    ,0 x  
 1  cos x  1  cos x  4 2 2
Sol. We have,

Sol. We have,  1  cos x  1  cos x 


tan 1  
 1  cos x  1  cos x 
 1  cos x  1  cos x 
tan 1    2 x x 
 1  cos x  1  cos x   2 cos  2 sin 2 
 tan 1  2 2 

 2 cos2 x  2 sin 2 x 
 2 2 
 2 x x
 2 cos  2sin 2 
 tan 1 2 2  x x 
 
 2 cos 2 x  2sin 2 x   2 cos 2  2 sin 2 
  tan 1  
2 2 
 2 cos x  2 sin x 
 2 2 

 x x
 x x   cos 2  sin 2 
 cos 2
 sin
2  tan 1
 
 tan 1     cos x  sin x 
 cos x x
 sin  2 2
 2 2

 3  x 3 
   x  2  2  2  4 
 x  x x   
 0  2  4  cos 2  0, sin 2  0  cos x   cos x , sin x  sin x 
   2 2 2 2 

 x x
 x 1
 cos 2
 sin
2
1  tan 2
 tan 
x

x
 tan 1    cos  sin
1  tan x  2 2
 2
 x
 1  tan 2 
 tan 1  
   x  1  tan x 
 tan 1  tan      2
  4 2 

   x 
 tan 1  tan    
 x   4 2 
 
4 2
 x
 
4 2
    x 
 0  x  2  4  4  2  2   3   x 
     x  2  2  4  2   4 
 
28 RELATION , FUNCTION & INVERSE TRIGONOMETRIC FUNCTIONS

EXERCISE - 1 : BASIC OBJECTIVE QUESTIONS

RELATION AND FUNCTION–II 7. Let f : R  R be a function such that f (x) = x3 – 6x2 + 11x – 6. Then

Revision of Concept of Functions, Domain and Range (a) f is one-one and into (b) f is one-one and onto
(c) f is many-one and into (d) f is many-one and onto
log 2  x  3 
1. The domain of f (x) = is 8. Let f  : R  R be a function such that
x 2  3x  2
f (x) = x3 + x2 + 3x + sin x. Then
(a) R – {–1, –2} (b) (–2, + ) (a) f is one-one and into (b) f is many-one and into
(c)  R – {–1, –2, –3} (d) (–3, + ) – {–1, –2} (c) f is one-one and onto (d) f is many-one and onto
2
2. The range of the function y = log3 (5 + 4x – x ) is 9. Let f be a function from R to R given by f (x) = 2x + |cos x|.
(a) (0, 2] (b) (–, 2] Then f is
(c) (0, 9] (d) none of these (a) one-one and into (b) one-one and onto
x f(x)
3. If e + e = e, then range of the function of f is (c) many-one and into (d) many-one and onto
(a) (–, 1] (b) (–, 1)
x2 – 4
(c) (1, ) (d) [1, ) 10. Let f be a function from R to R given by f (x) = .
x 2 1
x2  x  2
4. Range of the function f (x) = ; x  R is Then f (x) is.
x2  x 1
(a) one-one and into (b) one-one and onto
 11  (c) many-one and into (d) many-one and onto
(a) (1, ) (b)  1, 
 7
11. f (x) = x + x 2 is a function from R  R. Then f (x) is
 7  7 (a) injective (b) surjective
(c)  1,  (d)  1, 
 3  5 (c) bijective (d) none of these
Even & Odd Functions, Types of Functions 12. A function f : A  B, where A {x : – 1  x  1} and
2
5. Which of the following functions is an odd function : B = {y : 1  y  2} is defined by the rule y = f (x) = 1 + x .
Which of the following statement is true?
(a) f  x   1  x  x 2 – 1– x  x 2 (a) f is injective but not surjective
(b) f is surjective but not injective
 ax  1
(b)  f (x) = x  x (c) f is both injective and surjective
 a – 1 
 
(d) f is neither injective nor surjective

1 – x   x, if x is rational
(c)  f (x) = log   13. f x   and
1  x  0, if x is irrational
(d)  f (x) = k (constant)
0, if x is rational
g x   . Then, f – g is
6.  
The function f (x) = cos l og x + x  1
2
 is :  x, if x is irrational

(a) even (b) odd (a) one-one and into (b) neither one-one nor onto

(c) constant (d) None of these (c) many one and onto (d) one-one and onto
RELATION , FUNCTION & INVERSE TRIGONOMETRIC FUNCTIONS 29

14. If f : R  S, define by f (x) = sin x – 3 cos x + 1, is onto, 22. The period of the function f (x) = |sin 4x| + |cos 4x| is

then the interval of S is


 
(a) [0, 1] (b) [–1, 1] (a) (b)
2 8
(c) [0, 3] (d) [–1, 3]
15. If a function f : [2, )  B defined by f (x) = x2 – 4x + 5 is a 
(c) (d) none of these
bijection, then B is : 4
(a) R (b) [1, )
(c) [4, ) (d) [5, ) 23. Let f (x) = cos 3x + sin 3 x. Then f (x) is

Transformation of Graphs and Periodic Functions (a) a periodic function of period 2.
16. The number of real solutions of the equation ex = x is
(b) a periodic function of period 3 .
(a) 1 (b) 2
(c) 0 (d) none of these (c) not a periodic function

17. The number of real solutions of the equation log0.5 x = |x| is (d) none of these

(a) 1 (b) 2 24. Which one is not periodic ?


(c) 0 (d) none of these 2 2
(a) |sin 3x| + sin x (b) cos x + cos x
18. The number of real solutions of the equation 2
sin (ex) = 5x + 5–x is (c) cos 4x + tan x (d) cos 2x + sin x
2
(a) 0 (b) 1 25. The period of sin  is
2
(c) 2 (d) infinitely many (a)  (b) 
3
(c)  (d) /2
x 2
1 
19. The equation 2 sin . cos2 x = x + , 0 < x < has 26. Which one is not periodic
2 x 2
2
(a) one real solution (a) |sin 3x| + sin x (b) cos x  cos 2 x
(b) no real solution 2 2
(c) cos 4x + tan x (d) cos x + sin x
(c) infinitely many real solutions
Composite and Inverse, Functional Equation
(d) none of these
4 4
27. Let f  (x) be a function defined on [0, 1] such that
20. The period of the function f (x) = sin x + cos x is

  x x Q
(a)  (b) 
2 f x  
1  x, x  Q

(c) 2 (d) none of these
21. Which of the following function has period  ? Then for all x  [0, 1], fof (x) is

 2x   x  (a) a constant (b) 1 + x


(a) 2cos    3sin  
 3   3  (c) x (d) none of these

(b) |tan x| + cos 2 x 28. If f  x   2  x and g  x   1  2 x , then the domain of

    f [g (x)] is
(c) 4cos  2x    2sin  x  
 2  4 (a) (– , 1/2] (b) [1/2, )
(d) none of the above (c) (– , –3/2] (d) none of these
30 RELATION , FUNCTION & INVERSE TRIGONOMETRIC FUNCTIONS

29. Let [x] denote the greatest integer  x. If f (x) = [x] and 35. If f is a real valued function such that f (x + y) = f (x) + f (y)
and f (1) = 5, then the value of f (100) is
 8    8 
g(x) = |x|, then the value of f  g     g  f     is (a) 200 (b) 300
  5    5 
(c) 350 (d) 500
(a) 2 (b) –2
 1 
(c) 1 (d) –1 36. If 2f (x + 1) + f   = 2x and x  –1, then f (2) is equal to
 x 1
30. If f : R  R is given by
(a) –1 (b) 2
1, when x is rational
f x   , 5
 1, when x is irrational 5
(c) (d)
3 2

 
then ( fof ) 1  3 is equal to
1 1
37. If 3 f (x) + 5 f     – 3,  x(0)  R, then  f (x) is
(a) 1 (b) –1 x x
equal to :
(c) 3 (d) 0
1 3  1  3 
a x  a x (a)   5x  – 6  (b)  –  5x – 6 
31. The inverse of the function f  x   x is 16  x  16  x 
a  a x

(where codomain of f(x) is (–1, 1)) 1 3 


(c)   5x  6  (d) none of these
16  x 
1 1 x  1 1 x 
(a) log a   (b) log a   38. Which of the following functions are not identical ?
2 1 x  2 1 x 
x 1
(a)  f (x) = and g(x) =
 1 x  x2 x
(c) log a   (d) none of these
 1 x 
x2
32. Let f : [–1, ) R be given by f  (x) = (x + 1) –1, 2 (b)  f (x) = and g(x) = x
x
x > –1. Then f –1 (x), is :
(c) f (x) = ln x4 and g(x) = 4 ln x
(a) – 1  x 1 (d)  f (x) = ln (x – 1) (x – 2) and  g(x) = ln (x – 1) + ln (x – 2)
39. Let f : R  R be a function given by f (x + y) = f (x) + f (y) for
(b) – 1 – x 1
all x, y  R such that f (1) = a. Then, f (x) =
(c) does not exist because f is not one-one (a) ax (b) ax
(d) does not exist because f is not onto (c) xa (c) a + x

33. The inverse of the function y = [1 – (x – 3) ] is


4 1/7 40. Let f be a real valued function satisfying f (x + y) = f (x) f (y)
7 1/4 7 1/4 for all x, y R such that f (1) = a. Then, f (x) =
(a) 3 + (1 – x ) (b) 3 – (1 – x )
7 1/4 (a) ax (b) ax
(c) 3 – (1 + x ) (d) none of these
(c) xa (c) none of these
x 2
34. Let f (x) = , x   1 . The value of  for which  1 
x 1 41. If a f (x + 1) + bf    x, x 1, a  –b, then f (1) is equal
 x +1 
f (a) = a, (a  0) is
to
1 1
(a) 1  (b) (a) a + b (b) a2 – b2
a a
1
1 1 (c) (c) None of these
(c) 1  (d)  1 a+b
a a
RELATION , FUNCTION & INVERSE TRIGONOMETRIC FUNCTIONS 31

42. If f (x) = ax 2 + bx + c satisfies the identity


1
48. If tan(cos–1 x) = sin  cot 1  , then x is equal to :
f (x + 1) – f (x) = 8x + 3 for all x  R. Then, (a, b) =  2
(a) (2, 1) (b) (4, –1)
(c) (–1, 4) (c) (–1, 1) 5 5
(a)  (b) 
3 3
INVERSE TRIGONOMETRIC FUNCTIONS
5
Basic Results and Calculation (c)  (d) None of these
3

 5   2  49. If k  sin–1 x + cos–1 x + tan–1 x  K then :


43. If  = tan–1  tan  and  = tan–1  – tan  , then
 4   3 

(a) k = 0, K =  (b) k  0, K 
2
(a) 4 = 3 (b) 3 = 4

(c) k  , K   (d) None of these
7 2
(c)  – = (d) none of these
12
50. If  = sin–1 x + cos–1 x – tan–1 x, x  0, then the smallest
interval in which  lies, is given by :
1 
44. If sin x  , for some x  (–1, 1), then the value of
5  3 
(a)  (b)  0
2 4 4
cos–1 x is :
  
(c) 0    (d) 
3 5 4 4 2
(a) (b)
10 10
11
51. The equation 2cos1 x  sin 1 x  has :
7 9 6
(c) (d)
10 10 (a) no solution (b) only one solution
(c) two solutions (d) three solutions
45. Which one of the following is correct?
52. cos [tan–1 {sin (cot–1 x)}] is equal to :
–1 –1
(a) tan 1 > tan 1 (b) tan 1 < tan 1
(c) tan 1 = tan 1 –1
(d) None of the above x2  2 x2  2
(a) (b)
x2  3 x2 1
Properties Based

x2 1
 (c) (d) None of these
1  3  x2  2
46. The value of sin   sin     is :
 2  2  
20 20
53. If  sin 1 x i  10 , then  xi is equal to :
i 1 i 1
3 3
(a) (b)  (a) 20 (b) 10
2 2
(c) 0 (d) None of these

1 1 54. Range of f(x) = sin–1 x + tan–1 x + sec–1 x is


(c) (d) 
2 2
  3    3 
(a)  ,  (b)  , 
47. 2 –1 2 –1
sec (tan 2) + cosec (cot 3) is equal to 4 4  4 4 

(a) 5 (b) 13   3 
(c)  ,  (d) None of these
(c) 15 (d) 6 4 4 
32 RELATION , FUNCTION & INVERSE TRIGONOMETRIC FUNCTIONS

55. Range of f (x) = sin–1x + cot–1x + tan–1x is


1
60. The value of cos (2 cos–1 x + sin–1 x) at x  is :
5
(a) [0, ] (b)   , 
 2 
(a) 1 (b) 3

 
(c)  ,  (d) [–, ] 2 6
4  (c) 0 (d) 
5

3 5
56. The value of cot 1  sin 1 is :
4 13   
61. If x  ,  , then the value of
 2 2
1 63 12
(a) sin (b) sin 1
65 13
 tan x  1  3sin 2x 
tan 1    tan   is
65 5  4   5  3cos 2x 
1
(c) sin (d) sin 1
68 12
(a) x/2 (b) 2x
x 1 x 1  (c) 3x (d) x
57. If tan 1  tan 1  , then x is equal to :
x2 x2 4
1 1
62. sin 1 x  sin 1  cos 1 x  cos 1 is equal to
1 1 x x
(a) (b) 
2 2


5 1 (a)  (b)
(c) (d)  2
2 2

58. If 2 tan–1 (cos x) = tan–1 (2 cosec x), then the value of x is : 3


(c) (d) None of these
2
3 
(a) (b)
4 4
1 2
63. tan 1    tan 1   is equal to
 4
  9
(c) (d) None of these
3

59. The value of tan–1 (1) + tan–1 (0) + tan–1 (2) + tan–1 (3) is 1 3 1 1  3 
(a) cos 1   (b) sin  
equal to : 2 5 2 5

5
(a)  (b)
4 1  3 1
(c) ta n 1   (d) ta n 1  
2 5 2

(c) (d) None of these
2
RELATION , FUNCTION & INVERSE TRIGONOMETRIC FUNCTIONS 33

64. cot 1  
cos   tan 1  
cos   x, then sin x is equal
67.
1 1
cos 1    2 sin 1   is equal to
2
  2
to

 
 2  (a) (b)
(a) tan 2   (b) cot   6 3
2 2

2 
 (c) (d)
(c) tan  (d) cot   3 4
2

a b 
y 68. If tan 1    tan 1    , then x is equal to :
65. If cos–1 x – cos–1  , then 4x2 – 4xy cos  + y2 is x x 2
2

equal to (a) ab (b) 2ab


(a) –4 sin2  (b) 4 sin2  (c) 2ab (d) ab
(c) 4 (d) 2 sin 2 69. If cos–1 x > sin–1 x, then :

3 4 (a) x < 0 (b) –1 < x < 0


66. If cos 1  sin 1  cos1 x, then x is equal to
5 5
1 1
(c) 0  x  (d) 1  x 
(a) 0 (b) 1 2 2

(c) –1 (d) None of these



 1 
70. The value of  tan 1  1  r  r 2  is equal to :
r 0

 3
(a) (b)
2 4


(c) (d) None of these
4
34 RELATION , FUNCTION & INVERSE TRIGONOMETRIC FUNCTIONS

EXERCISE - 2 : PREVIOUS YEAR JEE MAINS QUESTIONS

RELATION AND FUNCTION–II    


1. Which one is not periodic ? (2002) (a) 0,  (b)  0, 
 2  2
2 2
(a) |sin 3x| + sin x (b) cos x + cos x
     
(c) cos 4x + tan x
2
(d) cos 2x + sin x (c)  – ,  (d)  – , 
 2 2  2 2
2
2. The period of sin  is (2002)
2
9. A real valued function f (x) satisfies the functional
(a)  (b)  equation f (x – y) = f (x)  f (y) – f (a – x) f (a + y) where a is
3
(c)  (d) /2 a given constant and f (0) = 1.
f (2a – x) is equal to (2005)
–1   x 
3. The domain of sin l og 3  3   is (2002) (a) f (x) (b) –f (x)
  
(c) f (–x) (d) f (a) + f (a – x)
(a) [1, 9] (b) [–1, 9]
10. Let R = {(3, 3), (6, 6), (9, 9), (12, 12), (6, 12), (3, 9),
(c) [–9, 1] (d) [–9, –1] (3, 12), (3, 6)} be a relation on the set A = {3, 6, 9, 12}. The
7–x
4. The range of the function f (x) = P x–3 is (2004) relation is (2005)
(a) {1, 2, 3, 4} (b) {1, 2, 3, 4, 5, 6} (a) reflexive and symmetric only
(c) {1, 2, 3} (d) {1, 2, 3, 4, 5} (b) an equivalence relation

If f : R  S, define by f (x) = sin x – (c) reflexive only


5. 3 cos x + 1, is onto,
then the interval of S is (2004) (d) reflexive and transitive only

(a) [0, 1] (b) [–1, 1] 11. Let W denotes the words in the English dictionary. Define
the relation R by R = {(x, y) W × W : the words x and y
(c) [0, 3] (d) [–1, 3]
have at least one letter in common}. Then, R is
6. The graph of the function y = f (x) is symmetrical about the (2006)
line x = 2, then (2004)
(a) reflexive, symmetric and not transitive
(a) f (x) = f (– x) (b) f (2 + x) = f (2 – x)
(b) reflexive, symmetric and transitive
(c) f (x + 2) = f (x – 2) (d) f (x) = – f (– x)
(c) reflexive, not symmetric and transitive
–1
sin  x – 3 (d) not reflexive, symmetric and transitive
7. The domain of the function f (x) = 2
is
9–x
 
12. The largest interval lying in   ,  for which the
(2004)  2 2
(a) [1, 2] (b) [2, 3]
2 x 
(c) [2, 3) (d) [1, 2) function f (x)  4 x  cos 1   1 + log (cos x) is
2 
8. Let f : (–1, 1)  B, be a function defined by
defined, is (2007)

–1
 2x 
f (x) = tan  ,
2  then f is both one-one and onto   
 1– x  (a) [0, ] (b)   , 
 2 2
when B is the interval (2005)
    
(c)   ,  (d)  0, 
 4 2  2
RELATION , FUNCTION & INVERSE TRIGONOMETRIC FUNCTIONS 35

13. Let R be the real line. Consider the following subsets of 17. Let R be the set of real numbers.
the plane R × R, S = {(x, y) : y = x + 1 and 0<x<2}. Statement I : A = {(x, y) R × R : y – x is an integer} is an
T = {(x, y) : x – y is an integer} equivalence relation on R.
Which one of the following is true ? (2008) Statement II : B = {(x, y) R ×R : x = y for some rational
(a) T is an equivalence relation on R but S is not number } is an equivalence relation on R.

(b) Neither S nor T is an equivalence relation on R (2011)

(c) Both S and T are equivalence relations on R (a) Statement I is true, Statement II is true;

(d) S is an equivalence relation on R but T is not Statement II is not a correct explanation for Statement I.

14. Let f : N  Y be a function defined as f(x) = 4x + 3 where (b) Statement I is true, Statement II is false.

Y = {y N : y = 4x + 3 for some x N}. (c) Statement I is false, Statement II is true.

Show that f is invertible and its inverse is (2008) (d) Statement I is true, Statement II is true;
Statement II is a correct explanation for Statement I.
y3 3y  4
(a) g(y)  (b) g(y)  18. Consider the following relation R on the set of real square
4 3
matrices of order 3.
–1
y3 y3 R = {(A, B) |A = P BP for some invertible matrix P}
(c) g(y)  4  (d) g(y) 
4 4 Statement I : R is an equivalence relation.
2 Statement II : For any two invertible 3×3 matrices M and
15. Let f (x) = (x+1) –1, x  –1
–1 –1 –1
–1 N, (MN) = N M (2011)
Statement I : The set {x : f(x) = f (x)} = {0, –1}
(a) Statement I is false, Statement II is true.
Statement II : f is a bijection. (2009)
(b) Statement I is true, Statement II is true;
(a) Statement I is false, Statement II us true.
Statement II is correct explanation for Statement I.
(b) Statement I is true, Statement II is true;
(c) Statement I is true, Statement II is true;
Statement II is a correct explanation for Statement I.
Statement II is not a correct explanation for Statement I.
(c) Statement I is true, Statement II is true,
(d) Statement I is true, Statement II is false.
Statement II is not a correct explanation for StatementI.
2
19. Let f be a function defined by f(x) = (x–1) + 1, (x1)
(d) Statement I is true, Statement II is false.
–1
Statement I : The set {x : f (x) = f (x)} = {1, 2}
16. Consider the following relations
–1
R = {(x, y) | x, y are real numbers and x = wy for some Statement II : f is bijection and f (x) = 1 + x  1, x  1
rational number w};
(2011)

 m p  (a) Statement I is false, Statement II is true.


S   ,  m, n, p and q are integers such that n,
 n q  (b) Statement I is true, Statement II is true;
Statement II is a correct explanation for Statement I.
q  0 and qm = pn}. Then, (2010)
(c) Statement I is true, Statement II is true,
(a) R is an equivalence relation but S is not an equivalence
relation Statement II is not a correct explanation for Statement I.
(b) Neither R nor S is an equivalence relation (d) Statement I is true, Statement II is false.

(c) S is an equivalence relation but R is not an equivalence 20. Let X = {1, 2, 3, 4, 5}. The number of different ordered pairs
relation (Y, Z) that can formed such that Y  X, Z  X and Y  Z is
empty, is (2012)
(d) R and S both are equivalence relations
(a) 52 (b) 35
(c) 25 (d) 53
36 RELATION , FUNCTION & INVERSE TRIGONOMETRIC FUNCTIONS

21. Let A and B two sets containing 2 elements and 4 elements


3 3
respectively. The number of subsets of A × B having 3 or (c) 2 3 (d)   2 3
2 2
more elements is (2013)
(a) 256 (b) 220 2 1
27. If f (x)  x  x  5, x  and g(x) is its inverse function,
(c) 219 (d) 211 2

1 then g  7  equals: (2014/Online Set–3)


22. If g is the inverse of a function f and f (x)  , then
1  x5
1 1
g’(x) is equal to : (2014) (a)  (b)
3 13
(a) 1 + {g (x)}5 (b) 1 + x5
1 1
1 (c) (d) 
(c) 5x4 (d) 3 13
1  {g(x)}5
28. Statement I : The equation (sin-1x)3 + (cos-1x)3 - a3  0
9 2
23. If f(x) is continuous and f  , then
2 9 1
has a solution for all a a 
32
 1  cos 3x 
lim f   is equal to (2014/Online Set–1)
x 0  x2  
Statement II : For any x  R, sin-1 x + cos-1 x = and
2
(a) 9/2 (b) 2/9
(c) 0 (d) 8/9  x 92
0   sin 1 x   2  (2014/Online Set–3)
 4 16
x x
3 4
24. If f (x)        1, x  R, then the equations f(x) = (a) Both statements I and II are true.
5 5
(b) Both statements I and II are false.
0 has: (2014/Online Set–1)
(c) Statement I is true and statement II is false.
(a) no solution (b) one solution
(d) Statement I is false and statement II is true.
(c) two solutions (d) more than two solutions
29. If non-zero real numbers vb and c are such that min
25. Let P be the relation defined on the set of all real numbers
f(x) > max g(x), where f(x) = x 2 + 2bx + 2c 2 and
such that
P = {(a, b): sec2 a - tan2 b = 1}. Then P is: c
g(x) = - x2 - 2cx + b2  x  R  ; then lies in the interval:
b
(2014/Online Set–1)
(a) reflexive and symmetric but not transitive. (2014/Online Set–4)

(b) reflexive and transitive but not symmetric.


 1 1 1 
(c) symmetric and transtive but not reflexive. (a)  0,  (b)  , 
 2 2 2 
(d) an equivalence relation.
Therefore, P is transitive  1 
26. Let f be a function defined on the set of real number such
(c) 
 2
, 2

(d)  2,  
11 30. The function f(x) = |sin 4x| + |cos 2x|, is a periodic function
taht for x > 0, f (x) = 3 sin x + 4 cos x. Then (x) at x  
6 with period : (2014/Online Set–4)
is equal to : (2014/Online Set–2) (a) 2 (b) 

3 3  
(a) 2 3 (b)   2 3 (c) (d)
2 2 2 4
RELATION , FUNCTION & INVERSE TRIGONOMETRIC FUNCTIONS 37

 43   1 1 x
31. The principal value of tan 1  cot  is : 36. The function f : R    ,  defined as f  x   ,
 4   2 2 1 x2

(2014/Online Set–4) is: (2017)


(a) invertible
3 3
(a)  (b) (b) injective but not surjective
4 4
(c) surjective but not injective
  (d) neither injective nor surjective.
(c)  (d)
4 4 37. Let f (x) = 210 . x + 1 and g (x) = 310 . x – 1.
If (fog)(x) = x, then x is equal to : (2017/Online Set–1)
 2x 
32. If f(x) = 2tan–1x + sin–1  2 
, x  1, then f(5) is equal
1  x  310  1 210  1
(a) (b)
to (2015/Online Set–1) 310  210 210  310

 1  65 
(a) (b) tan   1  310 1  210
2  156  (c) (d)
210  310 310  210

(c) 4 tan 1 5  (d) 


1 x2  1 x2 1
38. The value of tan 1 , | x | , x  0 , is
33. If f : R  S, defined by 2
1 x  1 x 2 2

f (x)  sin x  3 cos x + 1, is onto, then the interval of S is equal to : (2017/Online Set–1)
(2015/Online Set–2)  1 
(a) [0, 1] (b) [-1, 1] (a)  cos 1 x 2 (b)  cos 1 x 2
4 2 4
(c) [0, 3] (d) [-1, 3]
 1 
1  cos 1 x 2  cos 1 x 2
34. If f(x)  2f    3x, x  0, and (2016) (c)
4 2
(d)
4
x
39. A value of x satisfying the equation
S  x  R : f(x)  f(–x) ; then S :
sin [cot–1 (1 + x)] = cos [tan–1 x], is :(2017/Online Set–2)
(a) contains exactly one element
(b) contains exactly two elements. 1
(a)  (b) –1
(c) contains more than two elements. 2
(d) is an empty set.
1
1 (c) 0 (d)
35. For x  R, x  0, x  1, let f0  x   2
1 x

and fn 1  x   f0  fn  x   , n = 0, 1, 2, … Then the value x


40. The function f : N  N defined by f (x) = x  5   ,
5
8 2 3 where N is the set of natural numbers and [x] denotes the
of f100 (3) + f1    f2   is equal to:
3 3 2 greatest integer less than or equal to x, is :
(2016/Online Set–1) (2017/Online Set–2)
8 5 (a) one-one and onto.
(a) (b)
3 3 (b) one-one but not onto.
(c) onto but not one-one.
4 1
(c) (d) (d) neither one-one nor onto.
3 3
38 RELATION , FUNCTION & INVERSE TRIGONOMETRIC FUNCTIONS

41. If x = a, y = b, z = c is a solution of the system of linear 45. If the function f defined as


equations
1 k -1
x + 8y + 7z = 0 f  x  = - 2x ,x  0, is continuous at x =0, then the
x e -1
9x + 2y + 3z = 0
x+y+z=0 ordered pair (k, f(0)) is equal to :

such that the point (a, b, c) lies on the plane x + 2y + z = 6, (2018/Online Set–2)
then 2a + b + c equals : (2017/Online Set–2) (a) (3, 2) (b) (3, 1)
(a) –1 (b) 0
1 
(c) 1 (d) 2 (c) (2, 1) (d)  ,2 
3 
42. Let S = {     R × R : f  t  =   e t -   .
   2 × 3x   1
46. If f  x  = sin-1  x 
,thenf  -  equals:
sin  2 t  ,t  R is a differentiable function}. Then S is a  1+ 9   2

subset of : (2018/Online Set–1) (2018/Online Set–2)

(a) R ×  0,  (b)  0,   × R (a) - 3loge 3 (b) 3loge 3

(c) R ×  - ,0  (d)  -, 0  × R (c) - 3loge 3 (d) 3loge 3


43. Consider the following two binary relations on the set A 47. Let N denote the set of all natural numbers. Define two
={a, b, c} : binary relations on N as
R1={(c, a), (b, b), (a, c), (c, c), (b, c), (a, a)} and R1={( x, y)  N × N : 2 x + y =10} and
R2={(a, b), (b, a), (c, c), (c, a), (a, a), (b, b), (a, c)}. Then : R2={( x, y) N N : x + 2 y =10}. Then :
(2018/Online Set–1) (2018/Online Set–3)
(a) both R1 and R2 are not symmetric. (a) Range of R1 is {2, 4, 8}.
(b) R1 is not symmetric but it is transitive. (b) Range of R2 is {1, 2, 3, 4}.
(c) R2 is symmetric but it is not transitive. (c) Both R1 and R2 are symmetric relations.
(d) both R1 and R2 are transitive. (d) Both R1 and R2 are transitive relations.

x -1 48. Let A, B and C be three events, which are pair-wise


44. Let f : A  B be a function defined as f  x  = , where independent and E denotes the complement of an event
x -2
A = R - {2} and B = R - {1}. Then f is : E. If P  A  B  C  = 0 and P  C  > 0, then P   A  B |C 
(2018/Online Set–2) is equal to : (2018/Online Set–3)

3y - 1 (a) P  A  - P  B  (b) P  A  + P  B 
(a) Invertible and f -1 (y) = y - 1
(c) P  A  - P  B  (d) P  A  + P  B 

2y - 1
(b) Invertible and f -1 (y) = y - 1

2y + 1
(c) Invertible and f -1 (y) =
y -1

(d) Not invertible


39 RELATION , FUNCTION & INVERSE TRIGONOMETRIC FUNCTIONS

EXERCISE - 3 : ADVANCED OBJECTIVE QUESTIONS

SINGLE ANSWER QUESTIONS 5. The domain of definition of

1. If f (x) =
x –1
, then f (2x) is : f (x) = log2  – l og1/2 1  11 / 4  – 1 is
x 1   x  

(a) (0, 1) (b) (0, 1]


f  x  1 3 f  x  1 (c) [1, ) (d) (1, )
(a) (b)
f x3 f x3
1
log x
6. The domain of the function f (x) = x  is :
f x3 f x3 (a) (0, ) – {1} (b) (0, )
(c) (d)
f  x  1 3 f  x  1 (c) [0, ) (d) [0, ) – {1}

2. If 2 < x 2 < 3 then the number of positive roots of tan x + tan x


7. f x  is defined for :
3x
1 
   x  , {.} denotes the fractional part of x, is :
2

x 
1
(a) R+ (b) R  – 
 
(a) 0 (b) 1 3
(c) 2 (d) 3
   
3. Find all possible values of x satisfying : (c) R –  n  n I  (d) none of these
 2 

x  x – 2  8x  12

8. The domain of the function
 x – 2  x   x – 2  x 
 
 x ,
(where [ ] denotes the greatest function { } is fractional part). f (x) = cot–1  2 x R is :
 x –  x 2  
 
11
(a) 4,  (b) {4, 5}
where [ ] denotes greatest integer function.
 2
(a) R – {± n , n N} (b) R – { n , n  0, n Z}
 11  11 15  (c) R – {0} (d) none of these
(c) 5,  (d)  , 
 2 2 4
1
9. The domain of the function f (x) =
x – x  x 4 – x 1
12 9

 1 
4. The domain of the function f (x) =  – 1 is : is
 sin x 
(a) (–, –1) (b) (1, )

  (c) (–1, 1) (d) (–, )


(a)  2n, 2n   ,  n  I
 2 10. Let f (x) = x and g(x) = |x| for all x R. Then the function
(x) satisfying
(b) (2n , (2n + 1) )  n  I [(x) – f (x)]2 + [(x) – g (x)]2 = 0 is :
(c) ((2n – 1) , 2n)  n  I (a) (x) = x, x [0,  ) (b) (x) = x, x R

(d) None of the above (c) (x) = –x, x (–, 0] (d) (x) = x + |x|, x R
40 RELATION , FUNCTION & INVERSE TRIGONOMETRIC FUNCTIONS

11. If [x] denotes the greatest integer  x, then 18. If f (x + y) = f (x) . f (y) for all real x, y and f (0)  0 then the

2 2 1  2 2   2 98  f x
 3    3  99    3  99   ...   3  99  is equal to function g(x) =
1  f  x  
2 is
       

(a) 99 (b) 98 (a) even function (b) odd function


(c) 66 (d) 65 (c) odd if f (x) > 0 (d) neither even nor odd
12. The domain of the function 19. Let f : R  R be a function defined by

  1  
f  x   log 3  – log1/2  1  1/ 5  –1 is x2 – 8
  x   f (x) = , then f  is :
x2  2
(a) (–, 1) (b) (0, 1)
(a) one-one but not onto (b) one-one and onto
(c) (1, ) (d) none of these
(c) onto but not one-one (d) neither one-one nor onto
20. The function f  (x) = max {(1 – x), (1 + x), 2},
 1 
13. The domain of the function f (x) = 4 l og 3   is : x  (–, ) is equivalent to :
 cos x 

(a) (–, ) 1– x, x  –1



(a) f  x   2, –1 x 1
(b) (–, ) – {n| n  I} 1 x, x 1

  
(c) (–, ) –  2n 1 : n I 
 2 
1 x, x  –1
(d) none of the above 
(b) f  x   2, –1 x 1
14. The range of the function f (x) = |x – 1| + |x – 2|, –1 < x < 3, is 1– x, x 1

(a) [1, 3] (b) [1, 5]
(c) [3, 5] (d) none of these
1– x, x  –1

(c) f  x   1, –1 x 1
15. Let f  (x) = sin 2x  – cos 2x  (where [.] denotes the
1 x, x 1

greatest integer function), then range of f (x) will be :
(a) {0} (b) {1} (d) None of these
21. Let f (x) = min {x, x2}, for every x  R. Then :
(c) {0, 1} 
(d) 0,1, 2 
 x , x 1
  x 2 , x 1
  4 – x2   (a) f  x    x 2 , 0  x 1 (b) f  x   
16. The range of the function f (x) = sin  l og    is : x , x  0  x , x 1
  1– x   
  

(a) [0, 1] (b) (–1, 0)


 x 2 , x 1
(c) [–1, 1] (d) (–1, 1)  x , x 1 
(c) f  x    2 (d) f  x    x , 0  x 1
17. Let f  be a real valued function defined by  x , x 1 x 2 , x  0

ex – e– x
f (x) = then range of f (x) is : 22. Total number of solutions of the equation
ex  e x
sin x = |l ne| x|| are :
(a) R (b) [0, 1] (a) 8 (b) 10
(c) [0, 1) (d) [0, 1/2) (c) 9 (d) 6
RELATION , FUNCTION & INVERSE TRIGONOMETRIC FUNCTIONS 41

23. The number points (x, y), where curves |y| = ln |x| and 29. Let f (x) = x(2 – x), 0 < x < 2. If the definition of f is extended
(x –1)2 + y2 – 4 = 0 cut each other, is
over the set R – [0, 2] by f (x + 2) = f (x) then f is a
(a) 2 (b) 3
(a) periodic function of period 1
(c) 1 (d) 6
24. Let f (x) = max {1 + sin x, 1 – cos x, 1}  x  [0, 2] and g (b) non periodic function
(x) = max {1, | x –1|}  x  R. Then gof is :
(c) periodic function of period 2
(a) 2 (b) 1
(d) periodic function of period ½
1+sin x x0
(c)  (d) None of these
1– cos x x0  1 x 2 
30. The function f  (x) = cos  sin x  + sin–1   is
 2x 
 x   x 
25. The function  f (x) = sin   – cos   is
 n!    n 1 !  defined for :

(a) non periodic (a) x  {–1, 1} (b) x  [–1, 1]

(b) periodic, with period 2(n!) (c) x  R (d) x  (–1, 1)


(c) periodic, with period (n + 1)
(d) None of these  2– x  –1
31. If f (x) = cos–1    l og  3 – x   , then its domain
 4 
26. If the period of the function f (x) = sin   n  x  , where [n]
denotes the greatest integer less than or equal to n, is 2, is :
then :
(a) [–2, 6] (b) [–6, 2) (2, 3)
(a) 1 < n < 2 (b) 1 < n < 2
(c) [–6, 2] (d) [–2, 2) (2, 3]
(c) 1 < n < 2 (d) none of these
27. Let f be a real valued function with domain R satisfying 32. Let f : (–, 2]  (–, 4] be a function defined by
f (x + k) = 1 + [2 – 5 f (x) + 10 (f (x))2 – 10 ( f (x))3 f (x) = 4x – x2. Then f –1 (x) is :
4 5 1/5
+ 5 (f (x)) – (f (x)) ]
for all real x and some positive constant k, then the period (a) 2 – 4 – x (b) 2  4 – x
of the function f (x) is :
(a) k (b) 2k (c) 4–x (d) none of these
(c) non periodic (d) none of these
28. Let f be a real valued function with domain R satisfying 33. Let f  : R  R, g  : R  R be two functions given by

1 f (x) = 2x – 3, g(x) = x3 + 5. Then ( f og)–1 (x) is equal to :


0 <  f (x) < and for some fixed a,
2
1/ 3 1/ 2
 x –7   x 7 
1 2 (a)   (b)  
f (x + a) = – f  x  –  f  x    xR  2   2 
2

then the peroid of the function f (x) is :


1/ 3 1/ 3
(a) a (b) 2a  7  x–2
(c)  x –  (d)  
(c) non periodic (d) none of these  2  7 
42 RELATION , FUNCTION & INVERSE TRIGONOMETRIC FUNCTIONS

38. Complete solution set of the equation


 x , x 1
 |x2 – 1 + cos x| = |x2 – 1| + |cos x|
34. If f  x    , then f ( f (x)) is equal to
 2  x, x  1 belonging to (–2,) is

  
(a)  – ,    – 1, 1
 2 2
2  x , x  1  x, x  1
 
   3    
  (b)  – ,    – 1, 1   ,  
(a)  x , 1  x  1 (b)  2  x , 1  x  1  2 2 2 
 
 
 2  x , x 1  2  x , x 1
 3     
(c)  – , –    ,  
 2 2 2 

2x , x  1  – 3      
(d)  – 2,  – , – 1  1, 
  2   2   2


(c)  x , 1  x  1 (d) none of these

 1 1 1
39. Let f1 (n) = 1    ... , then
 2  x , x 1 2 3 n

f1(1) + f1(2) + f1(3) + ... + f1(n) is equal to :

 1– x  (a) n f1(n) – 1 (b) (n + 1) f1(n) – n


35. If 2 f  (x – 1) – f     = x, then f (x) is :
 x  (c) (n + 1) f1(n) + n (d) n f1(n) + n

1 1  1– x 4x
(a)  2 1  x   (b) 2  x – 1 – 40. If f  x   , then
1  x 
x
3 x 4 2

1  1   2   1996 
(c) x 2  3 (d) None of these f  f   .... f   is equal to
x2  1997   1997   1997 

36. The number of pairs, (x, y), x, y  R, satisfying


(a) 1997 (b) 998
4x2 – 4x + 2 = sin2 y and x2 + y2 < 3 are
(c) 0 (d) none of these
(a) 0 (b) 4
41. If  f (x) + 2  f (1 – x) = x2 + 2,  x R, then f (x) is given as:
(c) 2 (d) infinite
37. Let f be a function satisfying 2 2

(a)
 x – 1 (b)
 x – 2
2 f (xy) = ( f (x))y + ( f (y))x 3 3

n
(c) x2 – 1 (d) x2 – 2
and f (1) = k  1, then  f  r  is equal to :
r= 1 2 3 + 27
42. The minimum value of 2(x –3) is :

(a) kn –1 (b) kn (a) 1 (b) 2

(c) kn +1 (d) None of these (c) 227 (d) None of these


RELATION , FUNCTION & INVERSE TRIGONOMETRIC FUNCTIONS 43

x –1   2 
43. If f  x   then f (f (ax)) in terms of f (x) is equal to: 48. Let f  :   – ,  be a function defined as
x 1  3 3

f (x) = 3 sin x – cos x + 2. The f –1 (x) is given by


f  x  –1 f  x  1
(a) a  f  x  1 (b) a  f  x  – 1 –1  x – 2  
 x–2 
(a) sin –1  – (b) sin  
 2  6  2  6

f  x  –1 f  x  1 2  x–2
(c) (d) (c)  cos –1   (d) none of these
a  f  x  – 1 a  f  x  1 3  2 

49. The number of points (x, y), where the curves


44. A certain polynomial P(x), x R when divided by x – a,
|y| = l n |x| and (x – 1)2 + y2 – 4 = 0 cut each other, is
x – b, x – c leaves remainder a, b, c respectively. The
(a) 2 (b) 3
remainder when P(x) is divided by (x – a) (x – b) (x – c) is
(c) 1 (d) 6
(a, b, c and distinct).
50. f (x) = |x – 1|, f : R  R and g (x) = ex , g : [–1, )  R If the
+

(a) 0 (b) x function fog (x) is defined, then its domain and range
2
(c) ax + b – c (d) ax + bx + c respectively are.
(a) (0, ) & [0, ) (b) [–1, ) & [0, )
t
1+sin x  – 1 ,
45. If f  (x) = lim then range of f (x) is  1  1 
(c)  –1,   & 1 – ,  
t
t 
1+sin x  1  e 
(d)  –1,   &  – 1,  
e 

(a) {–1, 1} (b) {0, 1} y y



51. If f  x  , x –   xy , then f (m, n) + f (n, m) = 0
(c) {–1, 1} (d) {–1, 0, 1}  8 8 

(a) only when m = n (b) only when mn


 1 
46. Range of the function f  defined by f (x) =   (where (c) only when m = – n (d) for all m and n
 sin x 
1
[.] and {.} respectively denote the greatest integer and the 52. Let f (x) = xx, x   ,   and let g (x) be inverse of f (x),
e 
fractional part functions) is.
then g(x) must be.
(a) I, the set of integers
(a) x (1 + log x) (b) x (1 + log g(x))
(b) N, the set of natural numbers.
1
(c) W, the set of whole numbers (c) (d) non-existent
x 1+ l og g  x  
(d) {2, 3, 4, ....}
53. The range of the function f (x) = cos [x],
47. Period of the function f (x) = cos 2 {2x} + sin 2 {2x} is
for /2 < x < /2 contains.
(where {.} denotes the fractional part of x)
(a) {–1, 1, 0} (b) {cos 1, 1, cos 2}
 (c) {cos 1, –cos 1, 1} (d) [–1, 1]
(a) 1 (b)
2 54. Let A be a set containing 10 distinct elements, then the
total number of distinct function from A to A is
1 (a) 10
10
(b) 101
(c) (d) 
2 10 10
(c) 2 (d) 2 –1
44 RELATION , FUNCTION & INVERSE TRIGONOMETRIC FUNCTIONS

55. The number of roots of the question 62. Which of the following functions are periodic ?
1 + log2 (1 – x) = 2 is : –x (a) f (x) = sgn (e–x)
(a) 0 (b) 1
 1 if x is a rational number
(c) 2 (d) many (b) f  x   
0 if x is a irrational number
 sin 2 x  x   x   sin 4 x 
   
56. The period of the function 3 , where 8 8
(c) f  x   +
[.] denotes the greatest integer function, is 1+ cos x 1– cos x
(a) 1/2 (b) 1
 1  1
(c) 2 (d) non periodic (d) f  x    x     x –   2  – x 
 2  2
57. If the function f : [1,  )  [1,  ) is defined by
x(x–1) –1
(where [ ] denotes greatest integer function)
f (x) = 2 , then f  (x) is
63. The graph of the function y = f  (x) is as shown in the
x  x 1 figure. Then which one of the following graphs are correct?
1 1
(a)   (b)   1  1  4log 2 x  y
2 2

1 x
(c)   1  1  4log 2 x  (d) not defined –2 0 1 3
2

58. If the function f : R  R be such that f (x) = x – [x], where


y
[x] denotes the greatest integer less than or equal to x,
1
then f –1 (x) is

1 (a) |y| = sgn ( f (x)) x


(a) (b) [x] – x –2 –1 1 2 3
x – x
–1
(c) not defined (d) none of these
y
59. The complete solution set of sin–1 (sin 5) > x2 – 4x is
1
(a) x  2  9  2 (b) x  2  9  2
(b) |y| = sgn (–f (|x|)) x
–3 –2 –1 1 2 3
(c) x  9  2 (d) x  9  2

MULTIPLE CHOICE QUESTIONS


60.  is the period of the function y
1
1 sin x 
(a) (b) |sin x| + |cos x| X
cos x 1 cosec x  (c) |y| = | f  | x ||
–1
–3 1 3
–2 2
(c) sin 2x + cos 3x (d) cos (sin x) + cos (cos x)
61. Let R = {(x, y) : x, y  R, x2 + y2 < 25} y

4 2
R= {(x, y) : x, y  R, y > x } then 1
9
(d) y = xsgn (f (x)) x
–2 –1 1 2 3
(a) dom R  R= [–3, 3] (b) Range R  R [0, 4]
(c) Range R  R= [0, 5] (d) R  R defines a function
RELATION , FUNCTION & INVERSE TRIGONOMETRIC FUNCTIONS 45

64. Let f (x) = [x]2 + [x + 1] –3, where [x] = the greatest integer 70. Let R = {(x, y) : x, y  R. x 2 + y 2 < 25} and
< x. Then
 4 
(a) f (x) is a many-one and into function R    x , y  :x , yR , y  x 2  then is
 9 
(b) f (x) = 0 for infinite number of values of x
(c) f (x) = 0 for only two real values (a) dom R  R = [–3, 3]
(d) none of these (b) Range R  R [0, 4]
65. Let f (x) and g(x) be two real valued function given by, (c) Range R  R = [0, 5]
f (x) = – l nx and g(x) = e–x. Let h(x) = f (x) – x and
(d) R  R defines a function
m(x) = g(x) – x. Further more let the number of solutions of
h(x) = 0 and m(x) = 0 be a and b, then. 71. If f is an even function defined on the interval [–5, 5], then
(a) a  b (b) a = b the real values of x satisfying the equation
(c) a = 1 and b = 1 (d) None of these
 x 1 
66. Let f (x) be a function defined by, f (x) = f   , are
 x2 
4
f (x)  2sin2x + x + sin x – [x],  x  R then,
(a) f (x) is not a periodic function
–1  5 –3  5
(b) f (x) is periodic function with period 1 (a) (b)
2 2
(c) f (x + 10) = f (x – 10),  x  R
(d) least value of f (x) is 1,  x  R –2  5
(c) (d) none of these
2
67. Let f (x) be defined on [–, ] and is given by,

sin x –   x  0 Using the following passage, solve Q.72 to Q.74


f x  
cos x 0  x   Passage –1
Let  f : RR is a function satisfying f (2 – x) = f (2 + x) and
Let g (x) = f  |x| + |  f (x)|,  x  [–, ], then
f (20 – x) = f (x),  x  R. For this function f answer the
(a) g (x) = 0, has no real roots
following.
(b) g (x) = 0, has infinitely many real roots
72. If f (0) = 5, then minimum possible number of values of x
(c) g (x) = 0
satisfying f (x) = 5, for x  [0, 170] is.
(d) limit does not exist at x = 0
(a) 21 (b) 12
1
68. If f(x) = and g(x) is inverse of f (x) then (c) 11 (d) 22
1 x 3
73. Graph of y = f  (x) is
(a) g( f (x)) = 1 + x3 (b) g(x) = 1 + (g(x))3
(a) symmetrical about x = 18
1 (b) symmetrical about x = 5
(c) g   x   3 (d) g(x) = (1 – g(x))1/3
1  g  x   (c) symmetrical about x = 8

69. Let f (x) be invertible function and let f –1 (x) be its inverse. (d) symmetrical about x = 20
Let equation f { f –1(x)} =  f –1(x) has two real roots  and  74. If f (2)  f (6), then
(within domain of  f (x)), then
(a) fundamental period of f (x) is 1
(a)  f (x) = x, also have same two real roots.
(b) fundamental period of f (x) may be 1
(b)  f –1(x) = x, also have same two real roots.
(c) period of f (x) can’t be 1
(c)  f (x) = f –1(x), also have same two real roots.
(d) Area formed by (0, 0) (, f ()) and (, f ()) is 1 unit. (d) fundamental period of f (x) is 8
46 RELATION , FUNCTION & INVERSE TRIGONOMETRIC FUNCTIONS

Using the following passage, solve Q.75 to Q.77 79. ASSERTION : Let f (x) be a function satisfying
Passage –2
f (x – 1) + f (x + 1) = 2  f (x) for all x  R. Then f (x) is
A function f from a set X to Y is called onto, if for every
y  Y there exist x X such that f (x) = y. Unless the periodic with period 8.
contrary is specified, a real function is onto if it takes all REASON : For every natural number n there exists a
real values, otherwise it is called into function. Thus, if X
periodic function with period n.
and Y are finite sets, then f cannot be onto If Y contains
more element than X. (a) A (b) B
75. The polynomial function (c) C (d) D
n n–1 n–2
P (x) = a0x + a1x + a1x + .... + an , 80. ASSERTION : Every function defined in a symmetrical
where a0  0, is onto, for interval about origin can be uniquely expressed as the
(a) all positive integers n sum of an even function and an odd function.
(b) all even positive integers n
REASON : The set of values of parameter a for which the
(c) all odd positive integers n
(d) no positive integer  x2 
function f (x) defined as f (x) = tan (sin x) +   on the set
a 
x 2  2x  c
76. The function f (x) = is onto, if
x 2  4x  3c [–3, 3] is an odd function is, (9, ),

(a) 0 < c < 2 (b) 0 < c < 4 where [x] = greatest integer  x.

1 1 (a) A (b) B
(c) – c (d) 0 < c < 1
2 2 (c) C (d) D
77. Which of the following is not true ? 81. ASSERTION :
(a) A one-one function from the set {a, b, c} to {}
3 2 1
cosec1  cos 1  2cot 1  cot 1 7  cot 1 7.
is onto also. 2 3 7
(b) An onto function from an infinite set to a finite set
cannot be one-one.  
REASON : sin–1 x + cos–1 x =  tan–1 x + cot–1 x = 
 
(c) An onto function is always invertible.
(d) The function tan x and cot x are onto
1 1
ASSERTION REASON cosec–1 x = sin–1 , and for x > 0, cot–1 x = tan–1
x x
(A) If ASSERTION is true, REASON is true, REASON is a
correct explanation for ASSERTION. (a) A (b) B
(B) If ASSERTION is true, REASON is true, REASON is (c) C (d) D
not a correct explanation for ASSERTION.
82. ASSERTION : If a is twice the tangent of the arithmetic
(C) If ASSERTION is true, REASON is false.
mean of sin–1 x and cos–1 x, be is the geometric mean of tan
(D) If ASSERTION is false, REASON is true.
x and cot x, then x2 – ax + b = 0  x = 1
78. ASSERTION : The period of the function

 x   sin 1 x + cos 1 x 
f  x   sin  x   cos  cot [x] is 24. REASON : tan   1
4 2 3  2 
REASON : The period of sin x, cos x is 2 and period of
(a) A (b) B
f (x) + g(x) is the LCM of the periods of f (x) and g(x).
(a) A (b) B (c) C (d) D
(c) C (d) D
RELATION , FUNCTION & INVERSE TRIGONOMETRIC FUNCTIONS 47

x 2 x3 x : x is rational
1  
83. ASSERTION : sin  x  +  .... 86. Assertion : f  x   1– x : x is irrational ,
 2 4  

then f ( f (x)) = x

π  x4 x6  Reason : If a function y = f (x) is symmetric about


=  cos 1  x 2  + ... for 0 < |x| < 2 has a unique
2  2 4  y = x, then f ( f (x)) = x

(a) A (b) B
solution.
(c) C (d) D

1  (e) E
REASON : tan x  x +1  sin 1 x 2 + x +1  has no
2
87. Assertion : A function y = f  (x) is defined by
x2 – arc cos y = , then domain of
solution for  2 < x < 0.
 f (x) is R.

(a) A (b) B Reason : cos–1 y  [0, ].

(c) C (d) D (a) A (b) B

(c) C (d) D

84.
1 1
 1
ASSERTION : sin  tan tan x + tan 1 x    has

 (e) E

88. Assertion : The domains of


no non-zero integral solution.
f  x   cos  sin x   and
REASON : The greatest and least values of

7 3 3 g  x   sin  cos x  are same.


(sin–1 x)3 + (cos–1 x)3 are and respectively..
8 32
Reason :  0 < cos (sin x) < 1 and
(a) A (b) B
0 < sin (cos x) < 1.
(c) C (d) D (a) A (b) B
85. Assertion : If f (x) = sin x, then f (x) = cos x (c) C (d) D

f (–x) = – f (x) (e) E

 f (–x) = f (x) 89. Assertion : If f (x + y) + f (x – y) = 2 f (x) . f (y) 


x, y R and f (0)  0, then f (x) is an
Reason : The derivative of an odd function is
even function.
even and vice-versa.
Reason : If f (–x) = f (x), then f (x) is an even
(a) A (b) B
function.
(c) C (d) D
(a) A (b) B
(e) E (c) C (d) D

(e) E
48 RELATION , FUNCTION & INVERSE TRIGONOMETRIC FUNCTIONS

Match the Following 91. Match the column.

90. Match the column. Column–I Column–II

Column–I Column–II (A) Numbers of solution of 2[x] = x + 2 {x} (P) 1

(A) The number of possible values (P) 1 is (where [.], {.} greatest integer and

of k if fundamental period of least integer functions respectively)

(B) If x2 + y2 = 1 and maximum value of (Q) 2



sin–1 (sin kx) is , is
2
2
x + y is , then  is equal to
3
(B) Numbers of elements in the domain (Q) 2

of f (x) = tan–1 x + sin–1 x + sec–1 x is


 1  1
(C) Period of the function (R) 3 (C) f  x    f  x –  = f (x) for all (R) 3
 2  2

 x   x  xR, then period of f (x) is


f (x) = sin   . cos   is
 2  2 
Subjective
(D) If the range of the function (S) 4
x 5
f (x) = cos–1 [5x] is {a, b, c} & a + b + c 92. The function f (x) = takes exactly k integer values,
x 2 1


= , then  is equal to then k must be
2
93. Let S be the set of points (x, y) given by
(where [.] denotes greatest integer)
S = {(x, y)}; x2 + y2 – 10x + 16 = 0}

y
and f : S  R be given by f (x, y) =
x

 3 3
If range of f is  – ,  where k > 0. then k must be
 k k

94. If f (1) = 2 and  f (x + y) = f (x) f (y) for all natural numbers

n
x, y, the natural number a for which  f a  k =
k 1

16 (2n – 1), is

95. The number of integer values of m for which

f (x) = x3 – mx2 + 3x – 11 invertible is


49 RELATION , FUNCTION & INVERSE TRIGONOMETRIC FUNCTIONS

EXERCISE - 4 : PREVIOUS YEAR JEE ADVANCED QUESTIONS


2
RELATION AND FUNCTION–II 7. Let f (x) = (x + 1) –1, x > –1. Then the set
–1
{x :  f (x) = f (x)} is (1995)
Objective Questions I [Only one correct option]
1. Let  f (x) = |x – 1|. Then, (1983)  –3  i 3 –3 – i 3 
2 2 (a) 0, – 1, , 
(a) f (x ) = {f (x)} (b) f (x + y) = f (x) + f (y)  2 2 
(c) f (|x|) = | f (x)| (d) none of the above
(b){0,1, –1}
2. If f (x) = cos (log x), then
(c) {0, –1}
1 x  (d) 
f (x). f (y)   f    f  xy   has the value (1983)
2  y  8. The function f (x) = |px – q| + r |x|, x  (–, ) where
p > 0, q > 0, r > 0 assumes its minimum value only on one
1 point if (1995)
(a) –1 (b)
2 (a) p q (b) r q
(c) –2 (d) None of these (c) r p (d) p = q = r
3. The domain of definition of the function 2
9. If g {f (x)} = |sin x| and f {g (x)} = sin x   , then

y  x + 2 is (1983) (1998)
l og10 1 x 

(a) (–3, –2) excluding –2.5 (a) f  x   sin 2 x ,g  x   x


(b) [0, 1] excluding 0.5
(b) f  x   sin x ,g  x   x
(c) [–2, 1) excluding 0 (d) None of the above
4. Which of the following functions is periodic ? (1983) (c) f  x   x 2 ,g  x   sin x
(a) f (x) = x – [x], where [x] denotes the greatest integer (d) f and g cannot be determined
less than or equal to the real number x 10.
–1
If f (x) = 3x – 5, then f (x) (1998)
1 1
(b) f (x) = sin   for x  0, f (0) = 0
x (a) is given by 3x  5

(c) f (x) = x cos x


x 5
(d) None of the above (b) is given by
3
 x  a  x  b  (c) does not exist because f is not one-one
5. For real x, the function will assume all real
 x  c (d) none of these
values provided. (1984)
(a) a > b > c (b) a < b < c 11. If the function f : [1, )  [1, ) is defined by
x (x – 1) -–1
(c) a > c < b (d) a < c < b f (x) = 2 , then f (x) is (1999)
6. Let f (x) = sin x and g(x) = 1n |x|. If the ranges of the x  x 1
1 1
composition functions fog and gof  are R 1 and R 2 (a)  
2
(b)
2
1 1 4 l og 2 x 
respectively, then (1994)
(a) R1 = {u : –1 < u < 1}, R2 = {v : – < v < 0} 1
(b) R1 = {u : – < u < 0}, R2 = {v : –1 < v < 0} (c)
2

1 1 4 l og 2 x  (d) not defined
(c) R1 = {u : –1 < u < 1}, R2 = {v : – < v < 0}
(d) R1 = {u : –1 < u < 1}, R2 = {v : – < v < 0}
50 RELATION , FUNCTION & INVERSE TRIGONOMETRIC FUNCTIONS

12. Let f () = sin (sin + sin 3). Then f () (2000) x
(a) > 0 only when  > 0 19. Let f  x   , x   1 . Then, for what value of  is
x 1
(b) < 0 for all real  f [ f (x)] = x ? (2001)
(c) > 0 for all real  (a) (b)  2
2
(d) < 0 only when  < 0
(c) 1 (d) –1
13. The domain of definition of the function y (x) is given by 2
x y 20. Suppose f (x) = (x + 1) for x > – 1. If g(x) is the function
the equation 2 + 2 = 2, is (2000)
whose graph is reflection of the graph of f (x) with respect
(a) 0 < x < 1 (b) 0 < x < 1 to the line y = x, then g(x) equals (2002)
(c)  < x < 0 (d)  < x < 1
1
(a) – x –1, x  0 (b) 2
, x  –1
 1, x  0  x 1

14. Let g(x) = 1 + x – [x] and f (x) =  0, x  0 , then for all x, f
 1, x  0 (c) x 1, x  –1 (d) x –1, x  0

[g (x)] is equal to (2001) 21. Let function f  : R  R be defined by f (x) = 2x + sin x for
x R. Then  f  is (2002)
(a) x (b) 1
(a) one-to-one and onto
(c) f (x) (d) g (x)
(b) one-to-one but NOT onto
1 –1 (c) onto but NOT one-to-one
15. If f : [1, )  [2, ) is given by f (x) = x + then f  (x)
x
(d) neither one-to-one nor onto
equals. (2001)
x
x  x2 – 4 x 22. If f : [0, )  [0, ) and f (x) = , then f is (2003)
(a) (b) 1 x 2 1 x
2
(a) one-one and onto
2
x– x –4 (b) one-one but not onto
(c) (d) 1 x 2 – 4
4 (c) onto but not one-one
(d) neither one-one nor onto
l og 2  x  3 
16. The domain of definition of f  x   is
x 2  3x  2 x2  x  2
23. Range of the function f (x) = ; x  R is (2003)
x 2  x 1
(2001)
(a) R/ {–1, –2} (b) () 11
(a) (1, ) (b) 1, 
(c) R/ {–1, –2, –3} (d) ()/ {–1, –2}  7
2 2
17. Let f (x) = (1 + b ) x + 2bx + 1 and let m (b) be the minimum
value of f (x). As b varies, the range of m (b) is  7  7
(c) 1,  (d) 1, 
(2001)  3  5

 1 24. Domain of definition of the function


(a) [0, 1] (b) 0, 
 2 
f  x   sin 1  2x   for real valued x, is (2003)
6
1 
(c)  ,1 (d) (0, 1]
2   1 1  1 1
(a)   ,  (b)   , 
18. Let E = {1, 2, 3, 4} and F = {1, 2}, Then, the number of onto  4 2  2 2
functions from E to F is (2001)
(a) 14 (b) 16  1 1  1 1
(c)   ,  (d)   , 
(c) 12 (d) 8  2 9  4 4
RELATION , FUNCTION & INVERSE TRIGONOMETRIC FUNCTIONS 51
2
25. If f (x) = sin x + cos x, g(x) = x –1, then g( f (x)) is invertible 30. The function f : [0, 3]  [1, 29], defined by
in the domain. (2004) f (x) = 2x3 – 15x2 + 36x + 1, is (2012)
(a) one-one and onto
    
(a) 0,  (b)  – ,  (b) onto but not one-one
 2  4 4
(c) one-one but not onto
   (d) neither one-one nor onto
(c)  – ,  (d) [0, ]
 2 2 31. For any positive integer n, define f n : (0, )  R as

 x , if x is rational  1 
26. f x   f n (x)   nj1 tan 1   for all
0 , if x is irrational  1  (x  f ) (x  j  1) 
x  (0, ) (Here, the inverse trigonometric function tan–
0 , if x is rational
g x   .
 x , if x is irrational 1   
x assumes value in   ,  .) Then, which of the
 2 2
Then f –g is. (2005)
following statement(s) is (are) TRUE? (2018)
(a) neither one-one nor onto
(b) one-one and onto
(c) one-one and into
 
(a)  5j1 tan 2 f j (0)  55

(d) many one and onto 10


 '
 2
 
(b)  j1 1  f1 (0) sec f j  0   10
27. Suppose X and Y are two sets and f : X  Y is a function.
For a subset A of X, define f(A) to be the subset 1
–1 (c) For any fixed positive integer n, lim tan  f n (x)  
{f(a) : a A} of Y. For a subset B of Y, define f (B) to be x  n
the subset {x X : f(x) B} of X. Then which of the
(d) For any fixed positive integer n, xlim sec 2  f n (x)   1
following statements is true ? (2005) 

(a) f 1  f (A)   A for every A  X Assertion and Reason


For the following questions choose the correct answer
(b) f 1  f (A)   A for every A  X if and only if f(X) = Y from the codes (a), (b), (c) and (d) defined as follows.
(A) Statement I is true, Statement II is also true; Statement

1

(c) f f (B)  B for every B  Y II is the correct explanation of Statement I.
(B) Statement I is true, Statement II is also true; Statement

1

(d) f f (B)  B for every B  Y if and only if f(X) = Y II is not the correct explanation of Statement I.
28. Let S = {1, 2, 3, 4}. The total number of unordered pairs of (C) Statement I is true, Statement II is false.
disjoint subsets of S is equal to (2010) (D) Statement I is false, Statement II is true.
(a) 25 (b) 34 32. Let f (x) = 2 + cos x for all real x.
(c) 42 (d) 41 Statement I : For each real t, there exists a point c in
2
29. Let f (x) = x and g (x) = sin x for all x  R. Then, the set of [t, t + ] such that f  (c) = 0.
all x satisfying (fogogof) (x) = (gogof) (x), Because
where (fog) (x) = f (g(x)), is (2011)
Statement II : f (t) = f (t + 2) for each real t. (2007)
(a)  n ,n {0,1, 2, ...} (a) A (b) B
(c) C (d) D
(b)  n , n {1, 2, ...}
x2
 33. Statement I : The curve y = – + x + 1 is symmetric with
(c) + 2n, n  {..., –2, –1, 0, 1, 2, ....} 2
2 respect to the line x = 1.
(d) 2n, n  {..., –2, –1, 0, 1, 2, ....} Because
Statement II : A parabola is symmetric about its axis.
(2007)
(a) A (b) B
(c) C (d) D
52 RELATION , FUNCTION & INVERSE TRIGONOMETRIC FUNCTIONS

Passage
 x 
If a continuous f defined on the real line R, assumes positive 39. Let E1   x   : x  1and  0  and
 x  1 
and negative values in R, then the equation
f(x) = 0 has a root in R. For example, if it is known that a
   x  
continuous function f on R is positive at some point and E 2   x  E1 : sin 1  log e    is a real number  .
its minimum values is negative, then the equation f(x) = 0    x 1   
has a root in R.
x
(Here, the inverse trigonometric function sin–1x assumes
Consider f(x) = ke – x for all real x where k is real constant.
(2007)   
values in   , )
34.
x
The line y = x meets y = ke for k  0 at  2 2 

(a) no point (b) one point Let f : E1   be the function defined by


(c) two points (d) more than two points
 x 
35.
x
The positive value of k for which ke – x = 0 has only one f  x   log e  
 x 1 
root is
and g : E 2   be the function defined by
1
(a) (b) 1
e   x 
g  x   sin 1  log e   (2018)
(c) e (d) loge 2   x 1  
x
36. For k > 0, the set of all values of k for which ke – x = 0 has Columns A Column B
two distinct roots, is
1   e
(A) The range of f is (P)  , 

,
1   1  e   e  1 
 1
(a)  0,  (b)  , 1
 e e  (B) The range of g contains (Q) (0, 1)

1   1 1
(c)  ,   (d) (0, 1) (C) The domain of f contains (R)   , 
e   2 2

Match the Columns (D) The domain of g is (S)  , 0    0,  


Match the conditions/expressions in Column I with
statement in Column II.  e 
(T)  ,
37. Let the functions defined in Column I have domain  e  1 
() and range (). Objective Questions II [One or more than one correct option]
Column I Column II
x2
(A) 1 + 2x (p) onto but not one-one 40. If y = f (x) = , then (1984)
x 1
(B) tan x (q) one-one but not onto
(r) one-one and onto (a) x = f (y)
(s) neither one-one nor onto (b) f (1) = 3
(1992) (c) y increases with x for x < 1
(d) f is a rational function of x.
x 2  6x  5
38. Let f (x) = 2 . 2x 1
x  5x  6 41. If S is the set of all real x such that is positive,
2x 3  3x 2  x
Column I Column II
then S contains. (1986)
(A) If –1 < x < 1, then f (x) (p) 0 < f (x) < 1
satisfies  3  3 1
(a)   ,  (b)   , 
(B) If 1 < x < 2, then f (x) satisfies (q) f (x) < 0  2  2 4
(C) If 3 < x < 5, then f (x) satisfies (r) f (x) > 0
 1 1 1 
(D) If x > 5, then f (x) satisfies (s) f (x) < 1 (c)   ,  (d)  ,3 
 4 2 2 
(2007)
RELATION , FUNCTION & INVERSE TRIGONOMETRIC FUNCTIONS 53

42. Let g(x) be a function defined on [–1, 1]. If the area of the
  
equilateral triangle with two of its vertices at 46. Let f :   ,   R be given by
 2 2
(0, 0) & [x, g (x)] is 3 / 4 , then g (x) can be.
(1989) f (x) = (log (sec x + tan x))3.
Then (2014)
(a) g  x    1 x 2 (b) g  x   1 x 2 (a) f (x) is an odd function
(b) f (x) is a one-one function
(c) g  x    1 x 2 (d) g  x   1 x 2
(c) f (x) is an onto function
2 
43. If f (x) = cos [ ] x + cos [ ] x, where [x] stands for the (d) f (x) is an even function
greatest integer function, then (1991)
6 4
 47. If  = 3 sin   and  = 3 cos   , where the inverse
(a) f    1 (b) f () = 1
11
  9
2 trigonometric functions take only the principal values,
then the correct option(s) is (are) (2015)

(c) f () = 0 (d) f   1 (a) cos  > 0 (b) sin  < 0
4
(c) cos () > 0 (d) cos  < 0
b x sin(2x)
44. Let f : (0, 1)  R be defined by f (x) = 1 bx , where b is a 48. If g(x)  sin x
sin 1 (t)dt, then (2017)

constant such that 0 < b < 1. Then, (2011)    


(a) g     2 (b) g     2
(a) f is not invertible on (0, 1)  2  2

–1  1
(b) f  f  on (0, 1) and f  (b) =  
f  0 (c) g    2 (d) g    2
2 2

–1  1 Fill in the Blanks.


(c) f = f  on (0, 1) and f  (b) =
f  0
 2 2

–1
(d) f  is differentiable on (0, 1) 49. The values of f (x) = 3 sin  16  x  lie in the interval...

 
2
45. Let f : (–1, 1)  R be such that f (cos 4) = for (1983)
2  sec2 
 x2 
    50. The domain of the function f (x) = sin–1  l og 2  is given
 0,    ,  . Then, the value(s) of f  1  is/are  2 
 4 4 2 3
by...... (1984)
(2012)
 4  x2 
3 3 51. If f (x) = sin log  1  x  , then the domain of f (x) is...

(a) 1  (b) 1   
2 2
(1985)
2 2
(c) 1  (d) 1  52. There are exactly two distinct linear functions, .... and ...
3 3 which map {–1, 1} onto {0, 2}. (1989)
53. If f is an even function defined on the interval (–5, 5), then
four real values of x satisfying the equation
 x 1 
f (x) = f   are ...... (1996)
 x2
54 RELATION , FUNCTION & INVERSE TRIGONOMETRIC FUNCTIONS

      
54. If f (x) = sin2 x + sin2  x   + cos x cos  x   and 60. Given A   x:  x   and f (x) = cos x – x (1 + x) find
 3  3  6 3
f (A). (1980)
5
g   1, then (gof) (x) = ... (1996) 61. Let f be a one-one function with domain {x, y, z} and range
4
{1, 2, 3}. It is given that exactly one of the following
55. The number of real solutions of the equation statements is true and the remaining two are false.
   x i     x i 
f (x) = 1, f (y)  1, f (z)  2
 i
sin 1   x i 1  x       cos 1          x   –1
determine f  (1). (1982)
 i 1 
i 1  2   2  
  i 1  2  i 1 
62. Find the natural number a for which
 1 1
lying in the interval   ,  is _____ . n

 2 2
(Here, the inverse trigonometric functions sin–1 x and
 f a  k  16 2 1 ,
k 1
n

   where the functions f satisfies the relation


cos–1 x assume values in   ,  and 0,  , respectively)
 2 2 f (x + y) = f (x) f (y) for all natural numbers x, y and further
(2018) f (1) = 2. (1992)
True/False. tan x
63. Show that the value of , wherever defined never
56. n 1/n
If f (x) = (a – x ) , where a > 0 and n is a positive integer, the tan 3x
f [f (x)] = x. (1983)
1
2
lies between and 3. (1992)
x  4x  30 3
57. The function f (x) = 2 is not one-to-one. (1983)
x  8x  18

58. If f1 (x) and f2 (x) are defined on domains D1 and D2 64. A function f : IR  IR, where IR, is the set of real numbers,
respectively, then f1 (x) + f2 (x) is defined on D1  D2. x 2  6x – 8
(1988) is defined by f (x) = .
  6x – 8x 2
Analytical and Descriptive Questions.
Find the interval of values of  for which f is onto. Is the
functions one-to-one for  = 3 ? Justify your answer.
 x 1 x  3
59. Let y  (1996)
 x  2
65. Find the range of values of t for which
Find all the real values of x for which y takes real values.
(1980) 1  2x  5x 2   
2sin t  , t   ,  (2005)
3x 2  2x 1  2 2
66. Let f : [0, 4] [0, ] be defined by f (x) = cos–1 (cos x). The
number of points x [0, 4] satisfying the equation

10  x
f x
10
is (2014)
55 RELATION , FUNCTION & INVERSE TRIGONOMETRIC FUNCTIONS

ANSWER KEY
EXERCISE - 1 : BASIC OBJECTIVE QUESTIONS
RELATION AND FUNCTION–II
1. (d) 2. (b) 3. (b) 4. (c) 5. (a,c) 6. (a) 7. (d) 8. (c) 9. (b) 10. (c)
11. (d) 12. (b) 13. (d) 14. (d) 15. (b) 16. (c) 17. (a) 18. (a) 19. (b) 20. (b)
21. (b) 22. (b) 23. (c) 24. (b) 25. (b) 26. (b) 27. (c) 28. (d) 29. (d) 30. (b)
31. (b) 32. (d) 33. (d) 34. (c) 35. (d) 36. (c) 37. (b) 38. (b,c,d) 39. (b) 40. (a)
41. (d) 42. (b)
INVERSE TRIGONOMETRIC FUNCTIONS
43. (a) 44. (a) 45. (a) 46. (c) 47. (c) 48. (d) 49. (a) 50. (d) 51. (a) 52. (c)
53. (a) 54. (c) 55. (a) 56. (a) 57. (c) 58. (b) 59. (a) 60. (d) 61. (d) 62. (a)
63. (a,d) 64. (a) 65. (b) 66. (b) 67. (c) 68. (a) 69. (d) 70. (a)

EXERCISE - 2 : PREVIOUS YEAR JEE MAINS QUESTIONS


RELATION AND FUNCTION–II
1. (b) 2. (b) 3. (a) 4. (c) 5. (d) 6. (b) 7. (c) 8. (c) 9. (b) 10. (d)
11. (a) 12. (d) 13. (a) 14. (a) 15. (d) 16. (c) 17. (b) 18. (c) 19. (d) 20. (b)
21. (c) 22. (a) 23. (b) 24. (b) 25. (b) 26. (c) 27. (c) 28. (d) 29. (d) 30. (c)
31. (c) 32. (d) 33. (d) 34. (b) 35. (b) 36. (c) 37. (d) 38. (a) 39. (a) 40. (d)
41. (c) 42. (a) 43. (c) 44. (b) 45. (b) 46. (b) 47. (b) 48. (a)
EXERCISE - 3 : ADVANCED OBJECTIVE QUESTIONS
RELATION AND FUNCTION–II
1. (b) 2. (b) 3. (a) 4. (b) 5. (a) 6. (a) 7. (c) 8. (d) 9. (d) 10. (a)
11. (c) 12. (b) 13. (c) 14. (b) 15. (c) 16. (c) 17. (d) 18. (a) 19. (d) 20. (a)
21. (a) 22. (d) 23. (b) 24. (b) 25. (d) 26. (a) 27. (b) 28. (b) 29. (c) 30. (a)
31. (b) 32. (a) 33. (a) 34. (a) 35. (a) 36. (c) 37. (d) 38. (d) 39. (b) 40. (b)
41. (b) 42. (a) 43. (a) 44. (b) 45. (d) 46. (b) 47. (c) 48. (b) 49. (b) 50. (b)
51. (d) 52. (c) 53. (b) 54. (a) 55. (c) 56. (b) 57. (b) 58. (c) 59. (a) 60. (a)
61. (a, b, c) 62. (a, b, c, d) 63. (a, b) 64. (a, b) 65. (b, c) 66. (b, c, d) 67. (b, d) 68. (a, b) 69. (a, b, c) 70. (a, b, c)
71. (a, b) 72. (d) 73. (a) 74. (c) 75. (c) 76. (d) 77. (c) 78. (a) 79. (b) 80. (b)
81. (d) 82. (a) 83. (c) 84. (d) 85. (a) 86. (a) 87. (d) 88. (e) 89. (a)
90. (A  Q, B  Q, C  Q, D  R) 91. (A  R, B  R, C  R) 92. 0006 93. 0004 94. 0003 95. 0007

EXERCISE - 4 : PREVIOUS YEAR JEE ADVANCED QUESTIONS


RELATION AND FUNCTION–II
1. (d) 2. (d) 3. (c) 4. (a) 5. (d) 6. (d) 7. (c) 8. (c) 9. (a) 10. (b)
11. (b) 12. (c) 13. (d) 14. (b) 15. (a) 16. (d) 17. (d) 18. (a) 19. (d) 20. (d)
21. (a) 22. (b) 23. (c) 24. (a) 25. (b) 26. (b) 27. (d) 28. (d) 29. (a) 30. (b)
31. (d) 32. (b) 33. (a) 34. (b) 35. (a) 36. (a)
37. (A–q; B –r) 38. (A–p,r,s; B–q,s; C–q,s; D–p,rs) 39. (A–S; B–P; C–P) 40. (a,d) 41. (a,d) 42. (b,c)
 3 
43. (a,c) 44. (a) 45. (a,b) 46. (a,b,c) 47. (b, c, d) 48. (b,d) 49. 0 ,  50. Domain  [–2, –1]  [1, 2]
 2
1  5 3  5
51. (–2, 1) 52. y = x + 1 and y = – x + 1 53. x  , 54. 1 55. (2) 56. False
2 2
 1     3     –1
57. True 58. True 59. x  [–1, 2) [3, ) 60. f  A      1  ,  1   61. f  (1) = y
 2 3  3  2 6  6 
     3  
62. a = 3 64. 2 < < 14 65. t  ,    ,  66. (3)
  10   10 2 

You might also like